Крок 1 - Стоматологія 2013 (буклет)

1 / 200
Гіпоплазія емалі зумовлена домінантним геном, локалізованим в Х-хромосомі. Мати має нормальну емаль зубів, а у батька спостерігається гіпоплазія емалі. У кого з дітей буде виявлятися ця аномалія? Enamel hypoplasia is caused by a dominant gene located on the X chromosome. The mother has normal tooth enamel, and the father has enamel hypoplasia. Which of the children will have this anomaly ?

У половини дочок Half daughters

Тільки у синів Only sons

У всіх дітей All children have

У половини синів Half sons

Тільки у дочок Only in daughters

2 / 200
У 5-річної дівчинки на періанальних складках мати знайшла білих 'черв’ячків', які викликають у дитини свербіж і неспокій, і доставила їх до лабораторії. При дослідженні лікар побачив білих гельмінтів 0,5-1 см довжиною, ниткоподібної форми із загостреними кінцями, у деяких вони заокруглені. Який діагноз можна встановити? In a 5-year-old girl, the mother found white 'worms' on the perianal folds, which cause itching and restlessness in the child, and took them to the laboratory. During the examination the doctor saw white helminths 0.5-1 cm long, thread-like in shape with pointed ends, in some they are rounded. What diagnosis can be made?

Ентеробіоз Enterobiosis

Теніоз Taeniosis

Аскаридоз Ascariasis

Опісторхоз Opistorchosis

Дифілоботріоз Diphyllobotriosis

3 / 200
У фекаліях хворого з розладами травлення виявлені зрілі нерухомі членики ціп’яка; матка в них має 7-12 бічних відгалужень. Який це може бути вид гельмінта? In the feces of a patient with indigestion, mature immobile segments of a hookworm were found; the uterus has 7-12 side branches. What kind of helminth can this be?

Ціп’як ехінокока Echinococcus tapeworm

Стьожак широкий The strip is wide

Ціп’як озброєний Tipyak is armed

Ціп’як карликовий Dwarf creeper

Ціп ’як неозброєний Cip 'as unarmed'

4 / 200
У клітин, які здатні до поділу, відбуваються процеси росту, формування органел, їх накопичення, завдяки активному синтезу білків, РНК, ліпідів, вуглеводів. Як називається період мітотичного циклу, в якому відбуваються вказані процеси, але не синтезується ДНК: Cells that are capable of division undergo processes of growth, the formation of organelles, and their accumulation due to the active synthesis of proteins, RNA, lipids, and carbohydrates. What is the mitotic period called a cycle in which the specified processes occur, but DNA is not synthesized:

Анафаза Anaphase

Синтетичний Synthetic

Телофаза Telophase

Пресинтетичний Presynthetic

Премітотичний Premitotic

5 / 200
Відомо, що ген, відповідальний за розвиток груп крові системи MN, має два алельних стани. Якщо ген М вважати вихідним, то поява алельного йому гена N відбулася внаслідок: It is known that the gene responsible for the development of the blood groups of the MN system has two allelic states. If gene M is considered the original, then the appearance of the allelic gene N occurred as a result of:

Мутації Mutations

Кросинговеру Crossover

Реплікації ДНК DNA Replications

Комбінації генів Gene combinations

Репарації ДНК DNA Repairs

6 / 200
Проводиться каріотипування клітин здорової людини. В каріотипі знайдено дрібну акроцентричну непарну хромосому. Це може бути: Karyotyping of cells of a healthy person is carried out. A small acrocentric odd chromosome was found in the karyotype. It can be:

Хромосома групи С C chromosome

Хромосома групи В Chromosome group B

Хромосома групи А Chromosome group A

Y-хромосома Y chromosome

Х-хромосома X chromosome

7 / 200
До уролога звернувся чоловік 58-ми років зі скаргами на різкі болі при сечовиділенні та зменшення кількості сечі. Лікар припустив наявність сечокам’яної хвороби. В якій частині чоловічої уретри найбільш імовірне затримання каміння? A 58-year-old man consulted a urologist with complaints of sharp pains during urination and a decrease in the amount of urine. The doctor assumed the presence of urolithiasis. In which part of the male urethra the most likely retention of stones?

Pars pelvina Pars pelvina

Pars membranacea Pars membranacea

Pars intramuralis Pars intramuralis

Pars spongiosa Pars spongiosa

Pars prostatica Pars prostatica

8 / 200
Хворий з запаленням слизової язика (глосит) скаржиться на розлад смакової чутливості передніх двох третин язика. Ураженням якого нерва воно викликане? A patient with inflammation of the tongue mucosa (glossitis) complains of a disorder of taste sensitivity in the front two thirds of the tongue. What nerve is it caused by?

Язиковий Language

Малий кам’янистий Small rocky

Барабанна струна Drum String

Барабанний Drum

Язикоглотковий Lingopharyngeal

9 / 200
До відділення хірургічної стоматології надійшла новонароджена дівчинка, яка при смоктанні починала поперху-ватись. При обстеженні виявлена розщілина твердого піднебіння, яка була наслідком незрощення середнього лобового відростка з верхньощелепним відростком I-ої зябрової дуги. Розщілина знаходилась у піднебінні між: A newborn girl was admitted to the department of surgical stomatology, who began to cry when sucking. During the examination, a cleft of the hard palate was revealed, which was the result of non-union of the middle frontal process with the maxillary process 1st gill arch. The cleft was in the palate between:

Os incisivum et processus palatinus maxillae Os incisivum et processus palatinus maxillae

В длянц canalis incisivus In dylans canalis incisivus

Processus palatinus maxillae dextrae et sinistrae Processus palatinus maxillae dextrae et sinistrae

Processus palatinus maxillae et lamina horizontalis os palatinum Processus palatinus maxillae et lamina horizontalis os palatinum

Lamina horizontalis os palatinum dextrum et sinistrum Lamina horizontalis os palatinum dextrum et sinistrum

10 / 200
Хворий звернувся до лікаря зі скаргою на утруднення під час жування. При обстеженні виявлена атрофія правих скроневого i жувального м’язів. При відкриванні рота щелепа відхиляється ліворуч. Який нерв уражений? The patient turned to the doctor with a complaint of difficulty while chewing. The examination revealed atrophy of the right temporal and masticatory muscles. When opening the mouth, the jaw deviates to the left. Which nerve affected?

Верхньощелепний Maxillary

Щелепно-під’язиковий Maxillohyoid

Рухова частина нижньощелепного Move part of mandible

Нижній альвеолярний Inferior alveolar

Лицевий Face

11 / 200
Під час операції закриття природженої щілини піднебіння (уранопластики) при збиванні долотом крилоподібного гачка ушкоджено великий піднебінний канал. Виникла кровотеча. Яку артерію ушкоджено? During the operation to close the congenital cleft palate (uranoplasty) when the pterygoid hook was knocked down with a chisel, the large palatine canal was damaged. Bleeding occurred. Which artery was damaged?

Клиноподібна Wedge

Низхідна піднебінна Descending Palate

Висхідна піднебінна Eastern Palate

Висхідна глоткова Ascending pharyngeal

Задня верхня альвеолярна Posterior superior alveolar

12 / 200
У хворого виявлений фурункул у зовнішньому слуховому проході. Які з перелічених лімфатичних вузлів у першу чергу можуть відреагувати на запальний процес? The patient has a furuncle in the external auditory canal. Which of the listed lymph nodes can primarily react to the inflammatory process?

nodi lymphatici cervicales profundi nodi lymphatici cervicales profundi

nodi lymphatici retropharyngeales nodi lymphatici retropharyngeales

nodi lymphatici cervicales superficiales nodi lymphatici cervicales superficiales

nodi lymphatici mandibulares nodi lymphatici mandibulares

nodi lymphatici parotidei nodi lymphatici parotidei

13 / 200
У хворого внаслідок пошкодження шкіри в середній ділянці грудинно-ключичнососкоподібного м’яза виникла повітряна емболія. Яка вена шиї була травмована? The patient had an air embolism due to damage to the skin in the middle part of the sternocleidomastoid muscle. Which neck vein was injured?

Задня вушна Back auricle

Передня яремна Anterior jugular

Внутрішня яремна Internal jugular

Зовнішня яремна External jugular

Поперечна вена шиї Transverse jugular vein

14 / 200
До лікаря звернувся хворий з запаленням комірок решітчастої кістки (етмоїдит). При обстеженні виявлено порушення кровопостачання кістки. Гілками якої артерії кровопостачаю-ться решітчасті комірки в нормі? A patient with inflammation of the cells of the ethmoid bone (ethmoiditis) consulted a doctor. During the examination, a violation of the blood supply to the bone was revealed. Which artery's branches normally supply blood to the ethmoid cells?

A. cerebri anterior A. cerebri anterior

A. infraorbitalis A. infraorbitalis

A. ophthalmica A. ophthalmica

A. facialis A. facialis

A. transversa faciei A. transversa faciei

15 / 200
Юнака 18-ти років доставлено в лікарню з ознаками внутрішньої кровотечі. Під час гри в футбол отримав удар в ділянці лівого підребер’я. Ушкодження якого з органів, що проектуються в дану ділянку, може спричинити сильну кровотечу? An 18-year-old boy was taken to the hospital with signs of internal bleeding. While playing football, he was hit in the left hypochondrium. Damage to the organs that are projected into this area, can cause severe bleeding?

Flexura coli sinistra Flexura coli sinistra

Cauda pancreatis Cauda pancreatis

Fundus ventriculi Fundus ventriculi

Lien Lien

Ren sinistra Ren sinistra

16 / 200
Жінка звернулась до лікаря зі скаргами на набряклість та болючість нижньої кінцівки, припухлість вен та вузлів на медіальній поверхні стегна. Яка з вен уражена? A woman consulted a doctor with complaints of swelling and pain in the lower limb, swelling of veins and nodes on the medial surface of the thigh. Which of the veins is affected?

Підколінна Popular

Великогомілкова Velikohomilkova

Стегнова Thigh

Мала підшкірна Small subcutaneous

Велика підшкірна Large subcutaneous

17 / 200
У військкоматі під час УЗ-діагностики у юнака 19-ти років було виявлено опущення нирки. На рівні яких хребців розташовані нирки в нормі? At the Military Commissariat, a 19-year-old young man was diagnosed with a prolapsed kidney during an ultrasound scan. At which vertebrae are normal kidneys located?

XII грудний та I поперековий XII thoracic and I lumbar

XI грудний та III поперековий XI thoracic and III lumbar

IV-V поперекові IV-V lumbar

IX-X грудні IX-X December

IX-XII грудні IX-XII December

18 / 200
В гістологічному препараті визначається слизова оболонка, вкрита багатошаровим плоским незроговілим, місцями - багатошаровим плоским зроговілим епітелієм. До складу слизової оболонки входить також власна пластинка, м’язова пластинка відсутня. Визначте місце локалізації такої слизової оболонки: In the histological preparation, a mucous membrane is identified, covered with a multilayered flat non-keratinized, in some places - a multilayered flat keratinized epithelium. The mucous membrane also includes the lamina propria, the muscular plate is absent Determine the location of such a mucous membrane:

Ротова порожнина Oral cavity

Трахея Trachea

Тонка кишка Small intestine

Шлунок Stomach

Стравохід Esophagus

19 / 200
В гістологічному препараті молочного зуба дитини визначається гіпоплазія (недорозвинення) емалі. З діяльністю яких клітин пов’язані ці порушення? In the histological preparation of a child's milk tooth, hypoplasia (underdevelopment) of the enamel is determined. What cells are these disorders associated with?

Внутрішні емалеві клітини Internal enamel cells

Клітини проміжного шару емалевого органу Cells of the intermediate layer of the enamel organ

Клітини пульпи емалевого органу Enamel organ pulp cells

Одонтобласти Odontoblasts

Зовнішні емалеві клітини External enamel cells

20 / 200
У гістологічному препараті поздовжнього шліфу зуба в емалі визначаються чергування темних і світлих смуг шириною близько 100 мкм, орієнтованих перпендикулярно поверхні дентину. Назвіть дані структури емалі: In the histological preparation of the longitudinal section of the tooth, alternating dark and light bands with a width of about 100 microns, oriented perpendicular to the surface of the dentin, are determined in the enamel. Name these enamel structures:

Неонатальна лінія Neonatal line

Лінії Ретціуса Retzius Lines

Смуги Іунтера-Шрегера Unter-Schrager bands

Емалеві призми Enamel prisms

Перикіматії Perikimatias

21 / 200
Під час гаструляції у зародку недостатньо сформувався первинний Іензе-новський вузлик. Розвиток якого осьового органу загальмується? During gastrulation, the embryo did not sufficiently form the primary Ienzen nodule. The development of which axial organ is inhibited?

Мантійний шар нервової трубки Neural tube mantle layer

Нервові гребінці Nerve combs

Нервова трубка Neural tube

Нервовий жолобок Neural Groove

Хорда Horde

22 / 200
В результаті травми носа у чоловіка 32-х років пошкоджена слизова оболонка верхньої носової раковини. До яких наслідків це призвело? As a result of an injury to the nose of a 32-year-old man, the mucous membrane of the upper concha was damaged. What consequences did this lead to?

Недостатнє зігрівання повітря Insufficient air heating

Недостатнє зігрівання і зволоження повітря Insufficient air heating and humidification

Порушення нюху Impaired sense of smell

Порушення очищення повітря Air purification violation

Недостатнє зволоження повітря Insufficient air humidification

23 / 200
Під час мікроскопічного дослідження органу ЦНС виявлена сіра речовина, у якій нейрони утворюють три шари: молекулярний, гангліонарний і зернистий. Назвіть нейрони, що формують другий шар: During a microscopic examination of the CNS organ, gray matter was found in which neurons form three layers: molecular, ganglionic, and granular. Name the neurons that form the second layer:

Дрібні зірчасті Small stars

Кошикові Baskets

Клітини-зерна Grain cells

Великі зірчасті Big stars

Грушоподібні Pear

24 / 200
При утворенні зародка людини можна спостерігати появу у його складі порожнини, світлих дрібних бластомерів на периферії та темних великих бластомерів на одному з полюсів. Як називається зародок на цій стадії розвитку? During the formation of a human embryo, you can observe the appearance of a cavity, light small blastomeres on the periphery and dark large blastomeres at one of the poles. What is the name of an embryo at this stage of development ?

Морула Morula

Бластоциста Blastocyst

Гаструла Gastrula

Зигота Zygote

Зародковий диск Embryonic disc

25 / 200
На препараті м’якої мозкової оболонки виявляється судина, у стінці якої відсутня середня оболонка, зовнішня оболонка зрощена з оточуючою тканиною, внутрішня оболонка побудована із базальної мембрани та ендотелію. Що це за судина? On the preparation of the medulla, a vessel is found in the wall of which there is no middle membrane, the outer membrane is fused with the surrounding tissue, the inner membrane is made of basement membrane and endothelium. What kind of vessel is this?

Артерія змішаного типу Mixed type artery

Вена м’язового типу зі слабким розвитком м’язових елементів Muscular type vein with weak development of muscular elements

Артерія м’язового типу Muscular artery

Артеріола Arteriole

Вена волокнистого типу Fibrous vein

26 / 200
У першому критичному періоді в матковій трубі з невідомої причини в зародку відбулося розчинення оболонки запліднення. Яке ускладнення вагітності можливе в цьому випадку? During the first critical period in the fallopian tube, for an unknown reason, the embryo dissolved the fertilization membrane. What complication of pregnancy is possible in this case?

Утворення двох бластоцист Formation of two blastocysts

Повернення бластоцисти назад в ампулярну зону труби Return of the blastocyst back to the ampullary zone of the tube

Загибель зародка Death of embryo

Імплантація зародка в стінці труби Embroid implantation in the pipe wall

Інвагінація стінки бластоцисти Intussusception of blastocyst wall

27 / 200
У людини час кровотечі збільшений до 10 хвилин. Причиною цього може бути: In a person, the bleeding time is increased to 10 minutes. The reason for this may be:

Гіпопротеінемія Hypoproteinemia

Лімфопенія Lymphopenia

Еритропенія Erythropenia

Лейкопенія Leukopenia

Тромбоцитопенія Thrombocytopenia

28 / 200
У чоловіка 47-ми років за медичними показаннями була видалена слинна залоза, після чого різко зменшився вміст амілази в слині. Яка залоза була видалена? A 47-year-old man had his salivary gland removed for medical reasons, after which the content of amylase in saliva decreased sharply. Which gland was removed?

Підщочна Subcheek

Привушна Privushna

Ясенні Ashes

Підщелепна Submaxillary

Під’язикова Sublingual

29 / 200
В експерименті у тварини був перерізаний стовбур мозку, після чого у неї різко підвищився тонус м’язів-розгиначів (децеребраційна ригідність). Усунення впливу якої структури мозку на м’язи викликало цей стан? In the experiment, the animal's brain stem was cut, after which the tone of the extensor muscles increased sharply (decerebration rigidity). Elimination of the influence of which brain structure on m 'What caused this condition?

Червоне ядро Red Core

Блакитна пляма Blue spot

Смугасте тіло Striped body

Чорна субстанція Black substance

Сірий горб Gray hump

30 / 200
При обстеженні хворого з травматичним пошкодженням головного мозку виявлено, що він втратив дотикову чутливість. Який відділ кори мозку пошкоджений? During the examination of a patient with traumatic brain damage, it was found that he has lost touch sensitivity. Which part of the cerebral cortex is damaged?

Передня центральна звивина Anterior central gyrus

Лобна частка кори Frontal cortex

Задня центральна звивина Posterior central gyrus

Тім’яна частка кори Parietal lobe of the cortex

Потилична частка кори Occipital cortex

31 / 200
При підйомі пішки на 5-й поверх у людини підвищився артеріальний тиск. Причиною є збільшення: When walking up to the 5th floor, a person's blood pressure increased. The reason is the increase:

Об’єму циркулюючої крові Volume of circulating blood

В’язкості крові Blood viscosity

Кількості функціонуючих капілярів Numbers of functioning capillaries

Хвилинного об’єму крові Minute volume of blood

Вмісту іонів в плазмі крові Content of ions in blood plasma

32 / 200
При визначенні енерговитрат організму людини встановлено, що дихальний коефіцієнт дорівнює 1,0. Це означає, що у клітинах досліджуваного переважно окислюються: When determining the energy expenditure of the human body, it was established that the respiratory coefficient is equal to 1.0. This means that the cells of the subject are mainly oxidized:

Вуглеводи Carbohydrates

Білки Proteins

Вуглеводи та жири Carbohydrates and Fats

Жири Fats

Білки та вуглеводи Proteins and carbohydrates

33 / 200
У відповідь на розтягнення м’яза спостерігається його рефлекторне скорочення. З подразнення яких рецепторів починається ця рефлекторна реакція? In response to the stretching of the muscle, its reflex contraction is observed. The irritation of which receptors starts this reflex reaction?

Суглобові рецептори Joint receptors

Больові рецептори Pain receptors

Сухожилкові рецептори Іольджі Iolji tendon receptors

Дотикові рецептори Touch receptors

М’язові веретена Muscle spindles

34 / 200
При обробці перекисом водню слизової оболонки ротової порожнини хворого, кров пофарбувалась у коричневий колір замість піноутворення. При зниженні концентрації якого з перелічених ферментів це можливо? When treating the patient's oral mucosa with hydrogen peroxide, the blood turned brown instead of foaming. When the concentration of which of the listed enzymes is reduced, this is possible?

Псевдохолінестераза Pseudocholinesterase

Метгемоглобінредуктаза Methemoglobin reductase

Ацетилтрансфераза Acetyltransferase

Ілюкозо-6-фосфатдегідрогеназа Ilucose-6-phosphate dehydrogenase

Каталаза Catalase

35 / 200
Електрофоретичне дослідження сироватки крові хворого на пневмонію показало збільшення одної з білкових фракцій. Вкажіть її: Electrophoretic examination of the blood serum of a pneumonia patient showed an increase in one of the protein fractions. Specify it:

y-глобуліни y-globulins

Альбуміни Albumin

в-глобуліни b-globulins

а1-глобуліни a1-globulins

а2-глобуліни a2-globulins

36 / 200
У чоловіка 53-х років діагностовано сечокам’яну хворобу з утворенням уратів. Цьому пацієнту призначено алло-пурінол, який є конкурентним інгібітором ферменту: A 53-year-old man was diagnosed with urolithiasis with the formation of urates. This patient was prescribed allo-purinol, which is a competitive enzyme inhibitor:

Уратоксидаза Urate oxidase

Дигідроурацилдегідрогеназа Dihydrouracil dehydrogenase

Уриділтрансфераза Uridyltransferase

Уреаза Urease

Ксантиноксидаза Xanthine oxidase

37 / 200
Перетравлення білків у шлунку є початковою стадією розщеплення білків у травному каналі людини. Назвіть ферменти, які беруть участь в перетравлені білків у шлунку: Protein digestion in the stomach is the initial stage of protein breakdown in the human digestive tract. Name the enzymes involved in protein digestion in the stomach:

Хімотрипсин та лізоцим Chymotrypsin and lysozyme

Пепсин та гастриксин Pepsin and Gastrixin

Ентеропептидаза та еластаза Enteropeptidase and elastase

Трипсин та катепсини Trypsin and cathepsins

Карбоксипептидаза та амінопептида-за Carboxypeptidase and aminopeptide-by

38 / 200
Після загоєння рани на її місці утворився рубець. Яка речовина є основним компонентом цього різновиду сполучної тканини? After the wound healed, a scar formed in its place. What substance is the main component of this type of connective tissue?

Еластин Elastin

Хондроїтинсульфат Chondroitin sulfate

Кератансульфат Keratan sulfate

Гіалуронова кислота Hyaluronic acid

Колаген Collagen

39 / 200
При ряді гемоглобінопатій відбуваються амінокислотні заміни у а- і в -ланцюгах гемоглобіну. Яка з них характерна для серпоподібноклітинної анемії? In a number of hemoglobinopathies, amino acid substitutions occur in the α- and β-chains of hemoglobin. Which of them is characteristic of sickle cell anemia?

Аспартат-лізин Aspartate-Lysine

Метіонін-гістидин Methionine-histidine

Аланін-серин Alanine-serine

Гліцин-серин Glycine-serine

Глутамат-валін Glutamate-valine

40 / 200
Для лікування деяких інфекційних захворювань, що викликаються бактеріями, застосовуються сульфаніламідні препарати, що блокують синтез фактора росту бактерій. Який механізм дії цих препаратів? Sulfonamide drugs that block the synthesis of bacterial growth factor are used to treat some infectious diseases caused by bacteria. What is the mechanism of action of these drugs?

!нгібують всмоктування фолієвої кислоти !inhibit the absorption of folic acid

Беруть участь в окисно-відновних процесах Participate in redox processes

Є алостеричними інгібіторами ферментів Are allosteric enzyme inhibitors

Є антивітамінами п-амінобензойної кислоти Are antivitamins of p-aminobenzoic acid

Є алостеричними ферментами Are allosteric enzymes

41 / 200
Анаеробне розщеплення глюкози до молочної кислоти регулюється відповідними ферментами. Який фермент є головним регулятором цього процесу? Anaerobic breakdown of glucose to lactic acid is regulated by appropriate enzymes. Which enzyme is the main regulator of this process?

Енолаза Enolase

Глюкоза-6-фосфат-ізомераза Glucose-6-phosphate isomerase

Фосфофруктокіназа Phosphofructokinase

Альдолаза Aldolase

Лактатдегідрогеназа Lactate dehydrogenase

42 / 200
У хворого встановлено зниження синтезу вазопресину, що призводить до поліурії і, як наслідок, до вираженої дегідратації організму. Що з переліченого є провідним механізмом поліурії? The patient has been diagnosed with a decrease in vasopressin synthesis, which leads to polyuria and, as a result, severe dehydration of the body. Which of the following is the leading mechanism of polyuria?

Порушення канальцієвої реабсорбції іонів Na Disruption of canalician reabsorption of Na ions

Зниження канальцієвої реабсорбції води Reduction of canalician reabsorption of water

Порушення реабсорбції глюкози Glucose reabsorption disorder

Підвищення гідростатичного тиску Hydrostatic pressure increase

Зниження канальцієвої реабсорбції білку Decreased canalicular protein reabsorption

43 / 200
Катіонні глікопротеїни є основними компонентами слини привушних залоз. Які амінокислоти обумовлюють їх позитивний заряд? Cationic glycoproteins are the main components of parotid gland saliva. What amino acids determine their positive charge?

Лізин, аргінін, гістидин Lysine, arginine, histidine

Аспартат, глутамат, гліцин Aspartate, glutamate, glycine

Ілутамат, валін, лейцин Ilutamat, valine, leucine

Аспартат, аргінін, глутамат Aspartate, arginine, glutamate

Цистеїн, гліцин, пролін Cysteine, glycine, proline

44 / 200
У хлопчика 4-х років після перенесеного важкого вірусного гепатиту спо- стерігаються блювання, епізоди непритомності, судоми. У крові - гіперамо-ніємія. Порушення якого біохімічного процесу в печінці викликало такий стан хворого? A 4-year-old boy has vomiting, episodes of fainting, convulsions after severe viral hepatitis. Hyperammonemia in the blood. Violation of which biochemical process in liver caused this condition of the patient?

Порушення знешкодження амоніаку Ammonia neutralization violation

Активація декарбоксилювання амінокислот Activation of decarboxylation of amino acids

Пригнічення синтезу білків Inhibition of protein synthesis

Пригнічення ферментів трансаміну-вання Inhibition of transamination enzymes

Порушення знешкодження біогенних амінів Violation of neutralization of biogenic amines

45 / 200
У ході катаболізму гістидину утворюється біогенний амін, що має потужну судинорозширювальну дію. Назвіть його: In the course of histidine catabolism, a biogenic amine is formed, which has a powerful vasodilating effect. Name it:

Серотонін Serotonin

Норадреналін Noradrenaline

ДОФА DOFA

Дофамін Dopamine

Гістамін Histamine

46 / 200
Ті організми, які в процесі еволюції не створили захисту від H202, можуть жити лише в анаеробних умовах. Які з перелічених ферментів можуть руйнувати пероксид водню? Those organisms that in the process of evolution did not create protection against H202 can live only in anaerobic conditions. Which of the listed enzymes can destroy hydrogen peroxide?

Цитохромоксидаза, цитохром B5 Cytochrome oxidase, cytochrome B5

Оксигеназа та каталаза Oxygenase and catalase

Оксигенази та гідроксилази Oxygenases and hydroxylases

Пероксидаза та каталаза Peroxidase and catalase

Флавінзалежні оксидази Flavin-dependent oxidases

47 / 200
У пацієнта 50-ти років, що звернувся до стоматолога, був виявлений малиновий 'лакований'язик. У крові: знижена кількість еритроцитів і концентрації гемоглобіну, колірний показник 1,3, наявні ознаки мегалобластичного типу кровотворення, дегенеративні зміни білої крові. Яке захворювання крові було виявлене у хворого? A 50-year-old patient who went to the dentist was found to have a crimson 'lacquered' tongue. Blood: reduced number of erythrocytes and hemoglobin concentration, color index 1 ,3, there are signs of megaloblastic type of hematopoiesis, degenerative changes in white blood. What blood disease was detected in the patient?

Апластична анемія Aplastic anemia

Мієлоїдний лейкоз Myeloid leukemia

Іемолітична анемія Hemolytic anemia

Б12-фолієводефіцитна анемія B12-folate deficiency anemia

Залізодефіцитна анемія Iron deficiency anemia

48 / 200
У вагітної жінки розвинувся токсикоз із важкими повторними блюваннями упродовж доби. До кінця доби почали проявлятися тетанічні судоми і зневоднення організму. Який зсув кислотно-лужної рівноваги викликав зазначені зміни? A pregnant woman developed toxicosis with severe repeated vomiting throughout the day. By the end of the day, tetanic convulsions and dehydration of the body began to appear. What shift in the acid-base balance caused these changes?

Негазовий видільний алкалоз Nongaseous excretory alkalosis

Газовий ацидоз Gas acidosis

Негазовий видільний ацидоз Nongaseous excretory acidosis

Газовий алкалоз Gas alkalosis

Негазовий метаболічний ацидоз Nongaseous metabolic acidosis

49 / 200
Під час о6іду дитина поперхнулася i аспірувала їжу. Почався сильний кашель, шкіра і слизові ціанотичні, пульс прискорений, дихання рідке. Видих подовжений. Яке порушення зовнішнього дихання розвинулось у дитини? During the meal, the child sneezed and aspirated food. A strong cough began, the skin and mucous membranes were cyanotic, the pulse accelerated, breathing was rare. Exhalation was prolonged. What disorder of external breathing developed in child?

Дихання Чейна-Стокса Cheyne-Stokes Respiration

Дихання Кусмауля Kussmaul Breath

Стадія інспіраторної задишки при асфіксії Stage of inspiratory dyspnea in asphyxia

Стадія експіраторної задишки при асфіксії Stage of expiratory dyspnea in asphyxia

Дихання Біота Biot's Breath

50 / 200
Після занурення водолаза на глибину 60 м у нього з’явилися симптоми порушення функцій центральної нервової системи - збудження, ейфорія, ослаблення уваги, професійні помилки. Ці симптоми пов’язані з токсичною дією на нейрони: After the diver dived to a depth of 60 m, he developed symptoms of central nervous system dysfunction - excitement, euphoria, impaired attention, professional errors. These symptoms associated with a toxic effect on neurons:

Амоніаку Ammonia

Вуглекислого газу Carbon dioxide

Азоту Nitrogen

Кисню Oxygen

Лактату Lactate

51 / 200
У хворого у крові: лейкоцити -90 • 109/л. В лейкоцитарній формулі: е-0%, б- 0%, ю- 0%, п2%, с- 20%, лімфобласти - 1%, пролімфоцити - 2%, лімфоцити - 70%, м- 5%, клітини Боткіна-ГУмпрехта. Збільшені шийні, підщелепні лімфатичні вузли. Для якої патології характерна така картина? The patient's blood: leukocytes -90 • 109/l. In the leukocyte formula: e-0%, b- 0%, u- 0%, p2 %, c- 20%, lymphoblasts - 1%, prolymphocytes - 2%, lymphocytes - 70%, m- 5%, Botkin-Gumprecht cells. Enlarged cervical and submandibular lymph nodes. What pathology is this picture characteristic of?

Хронічний лімфолейкоз Chronic lymphocytic leukemia

Лімфогранульоматоз Lymphogranulomatosis

Інфекційний мононуклеоз Infectious mononucleosis

Гострий лімфолейкоз Acute lymphocytic leukemia

Хронічний мієлолейкоз Chronic myelogenous leukemia

52 / 200
У хворого з пародонтитом відмічається набряк ясен. Вони мають темно-червоний колір. Яке місцеве порушення кровообігу переважає в яснах хворого? A patient with periodontitis has swelling of the gums. They are dark red in color. What local blood circulation disorder prevails in the patient's gums?

Ішемія Ischemia

Артеріальна гіперемія Arterial hyperemia

Емболія Embolism

Тромбоз Thrombosis

Венозна гіперемія Venous hyperemia

53 / 200
Хвора 18-ти років скаржиться на загальну слабкість, швидку втомлюваність, пригнічений настрій. Має астенічний тип конституції. Ps- 68/хв., АТ-90/60 мм рт.ст. Встановлена первинна нейроциркуляторна артеріальна гіпотензія. Що є провідним фактором зниження артеріального тиску в хворої? An 18-year-old patient complains of general weakness, rapid fatigue, depressed mood. She has an asthenic constitution. Ps- 68/min., BP-90/60 mm Hg. Primary neurocirculatory arterial hypotension is established. What is the leading factor in lowering blood pressure in the patient?

Депонування крові в венах великого кола кровообігу Blood deposition in the veins of the great circulatory circle

Зниження тонусу резистивних судин Decreased tone of resistive vessels

Зменшення хвилинного об’єму крові Decrease in minute blood volume

Гіповолемія Hypovolemia

Зменшення серцевого викиду Decreasing cardiac output

54 / 200
У жінки, яка протягом 15-ти років страждала на виражену гіпертензію, останнім часом з’явились задишка, серцебиття, трохи знизився систолічний тиск. Який основний механізм виникнення у хворої серцевої недостатності? A woman who has suffered from severe hypertension for 15 years has recently developed shortness of breath, palpitations, and her systolic pressure has decreased slightly. What is the main mechanism of occurrence in a patient with heart failure?

Порушення проведення імпульсу по міокарду Violation of impulse conduction through the myocardium

Порушення регуляції серцевої діяльності Cardiac dysregulation

Перевантаження серця збільшеним опором викиду крові Overload of the heart with increased resistance to blood ejection

Перевантаження серця збільшеним об’ємом крові Overload of the heart with increased blood volume

Ушкодження міокарда Myocardial damage

55 / 200
У хворого з черепно-мозковою травмою відзначається дихання, яке характеризується дихальними рухами, що наростають за глибиною, а потім спадають, після чого настає недовготривала пауза. Для якого патологічного дихання характерний даний тип? A patient with a brain injury has breathing, which is characterized by respiratory movements that increase in depth and then decrease, after which there is a short-term pause. For which pathological breathing characteristic of this type?

Чейн-Стокса Chain-Stokes

Гаспінг-дихання Gasping Breath

Кусмауля Kusmaul

Апнейстичне Apneistic

Біота Biota

56 / 200
Після травматичного видалення зуба хворий скаржиться на сильний, тупий, без чіткої локалізації біль у ясні, підвищення температури тіла до 37,5oC. Діагностований альвеоліт. Який вид болю в цього хворого? After a traumatic tooth extraction, the patient complains of severe, dull pain in the gums without clear localization, an increase in body temperature to 37.5oC. Alveolitis is diagnosed. What kind of pain in this patient?

Епікритичний Epicritic

Вісцеральний Visceral

Відбитий Reflected

Фантомний Phantom

Протопатичний Protopathic

57 / 200
Дитина 10-ти років під час гри порізала ногу відламком скла і була направлений у поліклініку для введення про- типравцевої сироватки. З метою попередження розвитку анафілактичного шоку лїкувальну сироватку вводили за Безредкою. Який механізм лежить в основі подїбного способу гіпосенсибілі-зацїї організму? A 10-year-old child cut his leg with a piece of glass while playing and was sent to the polyclinic for administration of anti-tetanus serum. In order to prevent the development of anaphylactic shock, the therapeutic serum was administered according to Bezredka. What mechanism is at the basis of such a method of hyposensitivity of the body?

Блокування синтезу медіаторів опасистих клітин Blocking the synthesis of mast cell mediators

Стимуляція синтезу антиген-специфічних IgG2 Stimulation of the synthesis of antigen-specific IgG2

Зв’язування фасованих на опасистих клітинах IgE Binding of IgE packaged on mast cells

Стимуляція імунологічної толерантності до антигену Stimulation of immunological tolerance to antigen

Зв’язування рецепторів до IgE на опасистих клітинах IgE receptor binding on mast cells

58 / 200
У хворого із порушенням серцевого ритму при обстеженні на ЕКГ спостерігається: ЧСС50/хв., ритм синусовий, неправильний, інтервал PQ подовжений, періодичне випадіння комплексу QRS. Яке порушення серцевого ритму має місце? In a patient with a heart rhythm disorder, during an ECG examination, the following is observed: heart rate 50/min, sinus rhythm, irregular, PQ interval prolonged, periodic loss of the QRS complex. What kind of disorder heart rhythm taking place?

Блокада правої ніжки пучка Гіса Blockade of the right bundle branch

Повна AV-блокада Full AV block

Неповна AV-блокада II ст Incomplete AV-blockade II century

Синдром слабкості синусового вузла Weak sinus syndrome

Неповна AV-блокада I ст Incomplete AV blockade of the 1st century

59 / 200
Через деякий час після видалення зуба у пацієнта розвинулась атрофія країв зубної лунки. Назвіть цей вид атрофії: Some time after tooth extraction, the patient developed atrophy of the edges of the tooth socket. Name this type of atrophy:

Атрофія від дії фізичних та хімічних факторів Atrophy from physical and chemical factors

Дисфункціональна атрофія Dysfunctional atrophy

Атрофія внаслідок недостатності кровообігу Atrophy due to lack of blood circulation

Атрофія від тиску Atrophy from pressure

Нейротична атрофія Neurotic atrophy

60 / 200
На розтині трупа жінки 52-х років, яка тривалий час хворіла на хронічний гломерулонефрит, виявлено: значно зменшені, щільні, дрібнозернисті нирки, фібринозне запалення серозних і слизових оболонок, дистрофічні зміни паренхіматозних органів, набряк головного мозку. Яким ускладненням зумовлені описані зміни серозних оболонок і внутрішніх органів? The autopsy of a 52-year-old woman who suffered from chronic glomerulonephritis for a long time revealed: significantly reduced, dense, fine-grained kidneys, fibrinous inflammation of the serous and mucous membranes , dystrophic changes in parenchymal organs, cerebral edema. What complication is caused by the described changes in the serous membranes and internal organs?

ДВЗ -синдром DVZ syndrome

Сепсис Sepsis

Тромбоцитопенія Thrombocytopenia

Анемія Anemia

Уремія Uremia

61 / 200
У жінки 35-ти років діагностована дифтерія зіву. Померла при явищах гострої серцевої недостатності. На розтині: порожнини серця розширені в поперечнику, м’яз серця тьмяний, в’ялий, строкатий на розрізі, з жовтуватими ділянками під ендокардом. Який вид дистрофії виявлений у кардіоміоцитах? A 35-year-old woman was diagnosed with diphtheria of the pharynx. She died due to acute heart failure. At autopsy: the heart cavities are dilated in cross section, the heart muscle is dull, in 'bare, variegated on section, with yellowish areas under the endocardium. What type of dystrophy is found in cardiomyocytes?

Вуглеводна Hydrocarbon

Балонна Balloon

Гіаліново-крапельна Hyaline-drip

Гідропічна Hydropic

Жирова Fat

62 / 200
У молодого чоловіка видалено збільшений надключичний лімфовузол. Гістологічно: у центрі - осередок сирчастого некрозу, на периферії - вал епітеліоїдних клітин та лімфоцитів, серед яких зустрічаються клітини Пирогова-Лангханса. Діагностуйте процес у лімфатичних вузлах: An enlarged supraclavicular lymph node was removed from a young man. Histologically: in the center - a center of serous necrosis, on the periphery - a shaft of epithelioid cells and lymphocytes, among which Pirogov-Langhans cells are found . Diagnose the process in the lymph nodes:

Лепроматозний лімфаденіт Lepromatous lymphadenitis

Лімфогранульоматоз Lymphogranulomatosis

Метастаз Вірхова Virchow Metastasis

Туберкульозний лімфаденіт Tuberculous lymphadenitis

Сифілітичний лімфаденіт Syphilitic lymphadenitis

63 / 200
У чоловіка 35-ти років під язиком, в області вуздечки виявлена бляшка білого кольору, розмірами 0,8х0,5 см, що виступає над поверхнею слизової оболонки, шорсткувата. Мікроскопічно: епітелій потовщений із гіперке-ратозом, акантозом. У підепітеліаль-ній сполучній тканині круглоклітинна інфільтрація з розростанням колагенових волокон. Який найбільш імовірний діагноз? A 35-year-old man has a white plaque, 0.8x0.5 cm in size, protruding above the surface of the mucous membrane, rough . Microscopically: the epithelium is thickened with hyperkeratosis, acanthosis. In the subepithelial connective tissue, round cell infiltration with the growth of collagen fibers. What is the most likely diagnosis?

Папілома Papilloma

Хронічний виразковий стоматит Chronic ulcerative stomatitis

Плоскоклітинний рак із ороговінням Squamous cell carcinoma with keratinization

Лейкоплакія Leukoplakia

Афтозний стоматит Aphthous stomatitis

64 / 200
При гістологічному дослідженні стінки бронха і прилеглих ділянок легені виявлені пласти і тяжі атипового плоского епітелію. У клітинах - помірно виражені ознаки атипізму: поліморфізм, гіперхромія ядер, мітози. У центрі комплексів концентричні утворення рожевого кольору. Який найбільш імовірний діагноз? During histological examination of the wall of the bronchus and adjacent areas of the lung, layers and strands of atypical squamous epithelium were found. In the cells, there are moderately expressed signs of atypism: polymorphism, hyperchromia of nuclei, mitoses. In concentric formations of pink color in the center of the complexes. What is the most likely diagnosis?

Недиференційований рак Undifferentiated cancer

Скірозний рак Thorough cancer

Аденокарцинома Adenocarcinoma

Плоскоклітинний рак з ороговінням Squamous cell carcinoma with keratinization

Плоскоклітинний рак без ороговіння Squamous cell carcinoma without keratinization

65 / 200
При гістологічному дослідженні стінки кісти, що локалізується у ділянці верхньої щелепи, встановлено, що стінка кісти з середини вистелена багатошаровим плоским епітелієм з підлеглою грануляційною тканиною з лім-фолейкоцитарною інфільтрацією. Зовнішній шар представлений пухкою волокнистою сполучною тканиною, оточеною рубцевою фіброзною тканиною. Ці дані є підставою для встановлення такого діагнозу: During histological examination of the wall of the cyst localized in the area of the upper jaw, it was established that the wall of the cyst from the middle is lined with a multilayered flat epithelium with underlying granulation tissue with lympho-leukocyte infiltration . The outer layer is represented by loose fibrous connective tissue, surrounded by scar tissue. These data are the basis for establishing such a diagnosis:

Проста гранульома Simple granuloma

Кістогранульома Cystogranuloma

Кератокіста Keratocyst

Амелобластома Ameloblastoma

Епітеліальна гранульома Epithelial granuloma

66 / 200
Хворий 59-ти років з трансмураль-ним інфарктом міокарда лівого тттлу-ночка, помер від справжнього розриву серця - тампонади серця. Який процес у зоні інфаркту міг сприяти розриву серця? A 59-year-old patient with a transmural myocardial infarction of the left tttlu-nocturnal, died of a true rupture of the heart - cardiac tamponade. What process in the area of the infarction could have contributed to the rupture hearts?

Тромбоутворення Thrombosis

Аутолітичні процеси розплавлення тканини міокарда (міомаляція) Autolytic processes of myocardial tissue melting (myomalacia)

Формування рубця з витонченням лівого шлуночка Scar formation with thinning of the left ventricle

Заміщення сполучної тканини зони інфаркту (організація) Replacement of connective tissue of the infarct zone (organization)

- -

67 / 200
У хворого 77-ми років, що страждає на атеросклероз, з’явився біль у правій ступні. Ступня збільшена в розмірі, шкірні покриви чорного кольору, маце-ровані, демаркаційна зона невиражена. Який патологічний процес у ступні діагностовано? A 77-year-old patient suffering from atherosclerosis developed pain in the right foot. The foot is enlarged, the skin is black, macerated , the demarcation zone is not expressed. What pathological process is diagnosed in the foot?

Суха гангрена Dry gangrene

Нома Noma

Волога гангрена Wet gangrene

Коагуляційний некроз Coagulation necrosis

Секвестр Sequestration

68 / 200
Чоловік віком 42 роки помер при явищах вираженої інтоксикації та дихальної недостатності. На розрізі тканина легень у всіх відділах строката, з множинними дрібновогнищевими крововиливами та вогнищами емфіземи. Гістологічно у легенях: геморагічна бронхопневмонія з абсцедуванням, у цитоплазмі клітин епітелію бронхів еозинофільні та базофільні включення. Який найбільш імовірний діагноз? A 42-year-old man died due to symptoms of severe intoxication and respiratory failure. On cross-section, the lung tissue in all sections is variegated, with multiple small focal hemorrhages and foci of emphysema. Histologically, the lungs : hemorrhagic bronchopneumonia with abscessation, eosinophilic and basophilic inclusions in the cytoplasm of bronchial epithelial cells. What is the most likely diagnosis?

Респіраторно-синцитіальна інфекція Respiratory syncytial infection

Стафілококова бронхопневмонія Staphylococcal bronchopneumonia

Парагрип Paraflu

Аденовірусна інфекція Adenovirus infection

Грип Flu

69 / 200
У дівчинки 6-ти років протягом 5-ти місяців розвинулася деформація обличчя внаслідок симетричного збільшення об’єму обох кутів нижньої щелепи. Мікроскопічно: кісткова тканина щелепи заміщена волокнистою сполучною тканиною з великою кількістю судин, примітивними кістковими балочками. Яке захворювання найбільш імовірно? A 6-year-old girl developed facial deformity within 5 months due to a symmetrical increase in the volume of both corners of the lower jaw. Microscopically: the bone tissue of the jaw is replaced by fibrous connective tissue tissue with a large number of vessels, primitive bone beams. What disease is most likely?

Цементома Cementoma

Остеобластокластома Osteoblastoclastoma

Фіброзна дисплазія Fibrous dysplasia

Еозинофільна гранульома Eosinophilic granuloma

Херувізм Cherubism

70 / 200
При розтині тіла чоловіка, померлого від внутрішньокишкової кровотечі, в клубовій кишці спостерігається некроз групових і солітарних фолікулів, імбібіція мертвих тканин жовчю і кров’ю; в нижньому відрізку кишки -явища секвестрації та відторгнення некротичних мас з утворенням дефектів. Який з перелічених діагнозів найбільш вірогідний? At the autopsy of the body of a man who died of intra-intestinal bleeding, necrosis of group and solitary follicles was observed in the ileum, imbibition of dead tissues with bile and blood; in the lower segment of the intestine - phenomena of sequestration and rejection of necrotic masses with the formation of defects. Which of the listed diagnoses is the most likely?

Черевний тиф, стадія 'чи- стих'виразок Typhoid, stage 'pure' ulcer

Черевнотифозна форма сальмонельозу Terbotyphoid form of salmonellosis

Черевний тиф, стадія 'бру- дних'виразок Typhoid, stage of 'dirty' ulcers

Черевний тиф, стадія некрозу Typhoid fever, stage of necrosis

Хвороба Крона Crohn's disease

71 / 200
У хворого в обох щелепах рентгенологічно виявлено численні дефекти у вигляді гладкостінних округлих отворів. При гістологічному дослідженні - явища остеолізису і остеопорозу при явищах слабкого кісткоутворення. В сечі хворого знайдено білок БенсДжонса. Який найбільш імовірний діагноз? In both jaws of the patient, numerous defects in the form of smooth-walled round holes were detected x-ray. Histological examination revealed the phenomena of osteolysis and osteoporosis with weak bone formation. Protein was found in the patient's urine Bence Jones What is the most likely diagnosis?

Гострий мієлолейкоз Acute myelogenous leukemia

Мієломна хвороба Myeloma

Гострий недиференційований лейкоз Acute undifferentiated leukemia

Хронічний еритромієлоз Chronic erythromyelosis

Хронічний мієлолейкоз Chronic myelogenous leukemia

72 / 200
У хворого 23-х років після перенесеної ангіни розвинувся сечовий синдром (гематурія, протеїнурія, лейкоцитурія). В пункційному біоптаті нирок виявлена картина інтракапілярного проліфе- ративного гломерулонефриту, а еле-ктронномікроскопічно виявлені великі субепітеліальні депозити. Який патогенез цього захворювання? A 23-year-old patient developed a urinary syndrome (hematuria, proteinuria, leukocyturia) after a sore throat. A puncture biopsy of the kidneys revealed a pattern of intracapillary proliferative glomerulonephritis, and large subepithelial deposits were detected by electron microscopy. What is the pathogenesis of this disease?

Атопія, анафілаксія з утворенням IgE і фіксацією їх на опасистих клітинах Atopy, anaphylaxis with the formation of IgE and their fixation on mast cells

Клітинно обумовлений цитоліз Cell-induced cytolysis

Цитотоксична, цитолітична дія антитіл Cytotoxic, cytolytic effect of antibodies

Імунокомплексний механізм Immune complex mechanism

Гранулематоз Granulomatosis

73 / 200
В сироватці крові новонародженого знайдено антитіла до вірусу кору. Про наявність якого імунітету це може свідчити? Antibodies to the measles virus were found in the blood serum of a newborn. What kind of immunity does this indicate?

Природний активний Natural active

Штучний пасивний Artificial Passive

Природний пасивний Natural Passive

Штучний активний Artificial Active

Спадковий Hereditary

74 / 200
Обстежуючи дитину, лікар-стоматолог виявив нальот на мигдаликах і запідозрив атипову форму дифтерії. Був підготовлений мазок, зроблений посів на поживні середовища і визначена токсичність виділеної культури. Яка реакція використана для визначення токсигенності виділеного штаму дифтерійної палички? While examining the child, the dentist found a plaque on the tonsils and suspected an atypical form of diphtheria. A smear was prepared, cultured on nutrient media and the toxicity of the isolated culture was determined. What was the reaction used to determine the toxigenicity of the isolated diphtheria bacillus strain?

Реакція гемолізу Hemolysis reaction

Реакція зв’язування комплементу Complement binding reaction

Реакція аглютинації на склі Agglutination reaction on glass

Реакція кольцепреципітації Annular precipitation reaction

Реакція преципітації у гелі Gel precipitation reaction

75 / 200
Хворий надійшов до інфекційної лікарні на 8-й день зі скаргами на головний біль, нездужання, слабкість. Для серологічного дослідження взято кров. При проведенні реакція аглютинації Відаля встановлено, що вона позитивна в розведенні 1:200 з Одіагностикумом черевного тифу. Який діагноз можна встановити на підставі цього дослідження? The patient came to the infectious disease hospital on the 8th day with complaints of headache, malaise, weakness. Blood was taken for a serological test. When conducting the Vidal agglutination reaction, it was established that that it is positive in a 1:200 dilution with Odiagnosticum for typhoid fever. What diagnosis can be made based on this test?

Туберкульоз Tuberculosis

Черевний тиф Typhoid

Дизентерія Dysentery

Лептоспіроз Leptospirosis

Холера Cholera

76 / 200
З фекалій хворого виділені шигели Зонне. Які потрібно провести додаткові дослідження для встановлення джерела інфекції? Zonne's Shigella were isolated from the patient's feces. What additional studies should be conducted to establish the source of the infection?

Фаготипування виділеної чистої культури Phagotyping of selected pure culture

Антибіотикограма Antibiotic program

Реакція нейтралізації Neutralization reaction

Реакція зв’язування комплементу Complement binding reaction

Реакція преципітації Precipitation reaction

77 / 200
При мікроскопії мазка, взятого у хворого з гострим гнійним періоститом, лікар виявив грампозитивні бактерії, розташовані у вигляді скупчень, що нагадують виноградні грона. Які мікроорганізми мають дану морфологію? During microscopy of a smear taken from a patient with acute purulent periostitis, the doctor found gram-positive bacteria located in the form of clusters resembling grapes. What microorganisms have this morphology?'

Гриби роду Кандида Fungi of the genus Candida

Стрептококи Streptococci

Стафілококи Staphylococci

Тетракоки Tetracocci

Сарцини Sartsyn

78 / 200
Хворій жінці поставили клінічний діагноз 'гонорея'. Яке із перерахованих нижче досліджень можна застосувати для підтвердження діагнозу? A sick woman was clinically diagnosed with gonorrhea. Which of the following tests can be used to confirm the diagnosis?

Реакція іммобілізації Immobilization reaction

Проба з бактеріофагом Sample with bacteriophage

Реакція гемаглютинації Hemagglutination reaction

Зараження лабораторних тварин Infection of laboratory animals

Мікроскопія патологічного матеріалу Microscopy of pathological material

79 / 200
В інфекційне відділення з ознаками ураження печінки поступила хвора, стоматолог за фахом. Які методи лабораторної діагностики необхідно призначити для встановлення діагнозу 'вірусний гепатит В'? A patient, a dentist by profession, was admitted to the infectious department with signs of liver damage. What methods of laboratory diagnostics should be prescribed to establish a diagnosis of 'viral hepatitis B'?

Вірусологічне дослідження сечі Urine virological examination

Вірусологічне дослідження фекалій Virological examination of feces

Виявлення HBsAg в сироватці крові Detection of HBsAg in blood serum

Виявлення активності ферментів (альдолази, трансаміназ та ін.) Detection of enzyme activity (aldolase, transaminase, etc.)

Визначення функціональних проб печінки (білірубін і холестерин крові) Determination of functional liver tests (bilirubin and blood cholesterol)

80 / 200
У пацієнта з попереднім діагнозом 'сифіліс'лаборант взяв сироватку крові для постановки імунної реакції, яка основана на виявленні антитіл, які припиняють рух трепонем і призводять до їх загибелі. Яку реакцію було використано для діагностики? From a patient with a previous diagnosis of syphilis, a laboratory technician took blood serum to test an immune reaction, which is based on the detection of antibodies that stop the movement of treponems and lead to their death. What reaction was used for diagnosis?

Реакція преципітації Precipitation reaction

Реакція аглютинації Agglutination reaction

Реакція нейтралізації Neutralization reaction

Реакція іммобілізації Immobilization reaction

Реакція зв’язування комплементу Complement binding reaction

81 / 200
До стоматолога звернувся пацієнт із проявами кандидозу порожнини ро- та, який постійно рецидивує і не піддається лікуванню. При опитуванні з’ясувалось, що у пацієнта тривалий час відмічається підвищена температура, втрата маси тіла. Які дослідження треба провести у хворого? A patient with manifestations of candidiasis of the oral cavity, which constantly recurs and is not amenable to treatment, came to the dentist. During the survey, it was found that the patient has been experiencing increased temperature, loss of body weight. What tests should be performed on the patient?

Імунологічні та серологічні тести на ВІЛ-інфекцію Immunological and serological tests for HIV infection

Дослідити рівень специфічних антитіл до грибів Кандида Investigate the level of specific antibodies to Candida fungi

Перевірити стан гуморального імунітету Check the state of humoral immunity

Виділити чисту культуру збудника і дослідити чутливість до антибіотиків Isolate a pure culture of the pathogen and investigate sensitivity to antibiotics

Бактеріологічні дослідження на дисбактеріоз Bacteriological studies on dysbacteriosis

82 / 200
Чоловік 49-ти років був доставлений в гастроентерологічне відділення зі скаргами на нудоту, біль у епігастрії. Хворому було призначено фамотидин. Вкажіть молекулярний субстрат його дії: A 49-year-old man was brought to the gastroenterology department with complaints of nausea, pain in the epigastrium. The patient was prescribed famotidine. Specify the molecular substrate of its action:

М-холінорецептори M-cholinergic receptors

Н2-гістамінорецептори H2-histamine receptors

а1-адренорецептори a1-adrenoceptors

b-адренорецептори b-adrenoceptors

Na+Н+-АТФаза Na+H+-ATPase

83 / 200
В комплексному лікуванні гінгівіту хворому призначили препарат, який за хімічною будовою відноситься до похідних піримідину, стимулює лейкопоез, прискорює загоєння ран, підсилює ріст та розмноження клітин (процеси проліферації), виявляє протизапальну дію. Застосовується при лейкопеніях різного генезу, в стоматологічній практиці при запальних захворюваннях слизової оболонки ротової порожнини. Визначте препарат: In the complex treatment of gingivitis, the patient was prescribed a drug that chemically belongs to pyrimidine derivatives, stimulates leukopoiesis, accelerates wound healing, enhances cell growth and reproduction (proliferation processes) , has an anti-inflammatory effect. It is used for leukopenia of various genesis, in dental practice for inflammatory diseases of the mucous membrane of the oral cavity. Define the drug:

Метилурацил Methyluracil

Меркаптопурин Mercaptopurine

Метотрексат Methotrexate

Коамід Coamide

Ціанокобаламін Cyanocobalamin

84 / 200
У хворого на прийомі у стоматолога виник пароксизм тахікардії. Який з названих засобів слід використати для його купірування? The patient had a paroxysm of tachycardia at the dentist's appointment. Which of the named means should be used to stop it?

Дифенін Difenin

Ізадрин Izadrin

Атропін Atropine

Верапаміл Verapamil

Нітрогліцерин Nitroglycerin

85 / 200
Під час оперативного втручання анестезіолог для керованої гіпотонії за- стосував гангліоблокуючий засіб. Який препарат було призначено хворому в даному випадку? During the surgical intervention, the anesthesiologist used a ganglioblocking drug for controlled hypotension. What drug was prescribed to the patient in this case?

Пахікарпін Pachycarpine

Бензогексоній Benzohexonium

Пентамін Pentamine

Гігроній Hygronium

Пірілен Pyrylene

86 / 200
До поліклініки звернувся хворий зі скаргами на біль за грудниною, задишку і серцебиття. Після обстеження лікар діагностував у хворого ІХС і призначив верапаміл. Який механізм дії даного препарату? A patient came to the polyclinic with complaints of chest pain, shortness of breath, and palpitations. After the examination, the doctor diagnosed the patient with coronary heart disease and prescribed verapamil. What is the mechanism of action of this drug?

Блокує а-адренорецептори Blocks α-adrenoceptors

Блокує натрієві канали Blocks sodium channels

Блокує калієві канали Blocks potassium channels

Блокує кальцієві канали Blocks calcium channels

Блокує в-адренорецептори Blocks v-adrenoceptors

87 / 200
Хворий на хронічну серцеву недостатність протягом декількох місяців приймав в амбулаторних умовах диго-ксин. На певному етапі лікування у нього виникли симптоми передозування препарату. Яке явище лежить в основі розвитку цього ускладнення? A patient with chronic heart failure has been receiving digoxin as an outpatient for several months. At a certain stage of the treatment, he developed symptoms of drug overdose. What phenomenon underlies the development this complication?

Сенсибілізація Sensitization

Звикання Addiction

Функціональна кумуляція Functional cumulation

Матеріальна кумуляція Material accumulation

Тахіфілаксія Tachyphylaxis

88 / 200
У людини травматичне пошкодження грудинно-ключично-соскоподібного м’яза. Це призвело до зменшення величини: A person has a traumatic injury to the sternocleidomastoid muscle. This led to a decrease in size:

Функціональна залишкова ємність легенів Functional residual lung capacity

Резервний об’єм видиху Exhalation reserve volume

Резервний об’єм вдиху Inspiratory reserve volume

Дихальний об’єм Respiratory volume

Залишковий об’єм Remaining Volume

89 / 200
У дитини діагностовано порушення формування емалі і дентину зубів внаслідок зниженого вмісту іонів Ca2+ в крові. Який гормональний препарат можна застосувати для корекції даного стану? The child has been diagnosed with a violation of the formation of enamel and dentin of the teeth due to a reduced content of Ca2+ ions in the blood. What hormonal drug can be used to correct this condition?

Кальцитонін Calcitonin

Преднізолон Prednisone

Кортизон Cortisone

Тироксин Thyroxine

Соматотропін Somatotropin

90 / 200
У хворого після застудного захво- рювання виникло порушення виділення сльози. Який вегетативний вузол найбільше при цьому постраждав? After a cold, the patient developed a violation of tear secretion. Which vegetative node was most affected?

Війчастий Chilled

Піднижньощелепний Mandibular

Крилопіднебінний Phytopalatine

Вушний Ear

Під’язиковий Sublingual

91 / 200
У відділення інтенсивної терапії доставлено жінку 50-ти років з діагнозом: інфаркт міокарда. Активність якого ферменту крові буде найбільш підвищена протягом перших двох діб захворювання? A 50-year-old woman was brought to the intensive care unit with a diagnosis of myocardial infarction. The activity of which blood enzyme will be the most elevated during the first two days of the disease?

ЛДГ5 LDH5

ЛДГ4 LDH4

Аланінамінотрансфераза Alanine aminotransferase

Аланінамінопептидаза Alanine aminopeptidase

Аспартатамінотрансфераза Aspartate aminotransferase

92 / 200
На гістологічному препараті нирки в дистальному звивистому канальці виявляються клітини, які щільно прилягають до ниркового тільця. Базальна мембрана їх дуже тонка і не утворює складок. Ці клітини чутливі до зміни вмісту натрію у сечі та впливають на секрецію реніну юкстагломерулярни-ми клітинами. Які це клітини? On the histological preparation of the kidney in the distal convoluted tubule, cells are found that adhere closely to the renal corpuscle. Their basement membrane is very thin and does not form folds. These cells are sensitive to changes sodium content in urine and affect renin secretion by juxtaglomerular cells. What are these cells?

Клітини щільної плями Dense spot cells

Юкстагломерулярні клітини Juxtaglomerular cells

Ендотелій капілярів клубочка Endothelium of glomerular capillaries

Подоцити Podocytes

Мезангіальні клітини Mesangial cells

93 / 200
У хворого 26-ти років виявлений великий фурункул м’яких тканин обличчя біля кореня носа та нижньої повіки. Грізним ускладненням цього захворювання може бути розповсюдження інфекції венозними сполученнями цього регіону до пазух твердої мозкової оболонки. Яка з пазух найбільш імовірно може бути уражена? In a 26-year-old patient, a large furuncle of the soft tissues of the face near the root of the nose and the lower eyelid was detected. A formidable complication of this disease can be the spread of infection through the venous connections of this region to the sinuses of the dura mater. Which of the sinuses is most likely to be affected?

Верхня кам’яниста Upper stony

Верхня сагітальна Upper sagittal

Сигмоподібна Sigma

Потилична Occipital

Печериста Cave

94 / 200
Тварині, сенсибілізованій туберкуліном, внутрішньоочеревенно введений туберкулін. Через 24 години при лапа-ратомії виявлено венозну гіперемію та набряк очеревини. У мазках-відбитках з очеревини велика кількість лімфоцитів та моноцитів. Який патологічний процес у тварини? In an animal sensitized to tuberculin, tuberculin was injected intraperitoneally. After 24 hours, venous hyperemia and peritoneal edema were detected during laparotomy. In smear-prints from the peritoneum, a large number of lymphocytes and monocytes. What is the pathological process in the animal?

Алергічне запалення Allergic inflammation

Гнійне запалення Purulent inflammation

Серозне запалення Serous inflammation

Асептичне запалення Aseptic inflammation

Фібринозне запалення Fibrinous inflammation

95 / 200
У чоловіка 35-ти років через 30 хвилин після автомобільної аварії виявлена масивна травма нижніх кінцівок без значної зовнішньої крововтрати. По-страждалий знаходиться у збудженому стані. Який компонент патогенезу травматичного шоку є у пацієнта провідним і потребує негайного корегування? 30 minutes after a car accident, a 35-year-old man was found to have a massive injury to the lower extremities without significant external blood loss. The victim is apparently in an excited state. What component of pathogenesis is the patient leading to traumatic shock and needs immediate correction?

Порушення функції органів Organ dysfunction

Біль Pain

Внутрішня крововтрата Internal blood loss

Інтоксикація Intoxication

Внутрішня плазмовтрата Internal plasma loss

96 / 200
У процесі звапнування міжклітинної речовини кісткової тканини вздовж колагенових волокон відкладаються кристали гідроксиапатиту. Для реалізації цього процесу необхідна присутність у міжклітинній речовині лужної фосфатази. Яка клітина продукує цей фермент? In the process of calcification of the intercellular substance of bone tissue, hydroxyapatite crystals are deposited along the collagen fibers. This process requires the presence of alkaline phosphatase in the intercellular substance. Which cell produces this enzyme?

Остеобласт Osteoblast

Остеокласт Osteoclast

Хондробласт Chondroblast

Хондроцит Chondrocyte

Остеоцит Osteocyte

97 / 200
Хвора звернулась до лікаря зі скаргами на болі та обмеження рухів у колінних суглобах. Який з нестероїдних протизапальних засобів краще призначити, враховуючи наявність в анамнезі хронічного гастродуоденіту? The patient turned to the doctor with complaints of pain and limitation of movement in the knee joints. Which of the non-steroidal anti-inflammatory drugs is better to prescribe, taking into account the presence of chronic gastroduodenitis in the anamnesis?

Промедол Promedol

Целекоксиб Celecoxib

Парацетамол Paracetamol

Кислота ацетилсаліцилова Acetylsalicylic acid

Диклофенак-натрій Diclofenac sodium

98 / 200
Депресії, емоційні розлади є наслідком нестачі у головному мозку нора-дреналіну, серотоніну та інших біогенних амінів. Збільшення їх вмісту у си-напсах можна досягти за рахунок антидепресантів, які гальмують активність такого ферменту: Depression, emotional disorders are the result of a lack of noradrenalin, serotonin and other biogenic amines in the brain. Increasing their content in synapses can be achieved with antidepressants, which inhibit the activity of such an enzyme:

Оксидаза D-амінокислот D-amino acid oxidase

Діамінооксидаза Diamine oxidase

Моноамінооксидаза Monoamine oxidase

Фенілаланін-4-монооксигеназа Phenylalanine-4-monooxygenase

Оксидаза L-амінокислот L-amino acid oxidase

99 / 200
Хворому 30-ти років для лікування пневмонії лікар на 3 дні призначив антибіотик з групи азалідів, що має бактерицидну дію, тривалий ефект, здатність зв’язуватись з фагоцитами і накопичуватись у вогнищах інфекції. Який препарат було призначено хворому? A 30-year-old patient was prescribed an antibiotic from the azalide group for 3 days to treat pneumonia, which has a bactericidal effect, a long-lasting effect, the ability to bind to phagocytes and accumulate in foci of infection. What drug was prescribed to the patient?

Ізоніазид Isoniazid

Еритроміцин Erythromycin

Ципрофлоксацин Ciprofloxacin

Бензилпеніциліну натрієва сіль Benzylpenicillin sodium salt

Азитроміцин Azithromycin

100 / 200
З метою оцінки адаптації до фізичного навантаження лікар провів обстеження робітників після виконання важкої праці. Які зміни в загальному аналізі крові можна виявити при цьому? In order to assess adaptation to physical exertion, the doctor examined the workers after performing heavy work. What changes in the general blood analysis can be detected at the same time?

Анемія Anemia

Зсув лейкоцитарної формули вліво Shift of leukocyte formula to the left

Лейкопенія Leukopenia

Гіпоальбумінемія Hypoalbuminemia

Перерозподільчий лейкоцитоз Redistributive leukocytosis

101 / 200
Серед лімфоцитів розрізняють популяцію клітин, що мають мембранні рецептори до IgM, вони активуються під впливом специфічних антигенів, мітотично розмножуються, диференціюються у плазматичні клітини, що виробляють антитіла (імуноглобуліни). Як називаються ці клітини? Among the lymphocytes, a population of cells with membrane receptors for IgM is distinguished, they are activated under the influence of specific antigens, multiply mitotically, and differentiate into plasma cells that produce antibodies (immunoglobulins ). What are these cells called?

Т-лімфоцити пам’яті Memory T lymphocytes

Т-лімфоцити-кілери Killer T-lymphocytes

В-лімфоцити B-lymphocytes

- -

Т-лімфоцити-супресори Suppressor T lymphocytes

102 / 200
У ході клінічного обстеження пацієнта виявлено збільшення щитоподібної залози (зоб), підвищення основного обміну, втрата маси тіла, порушення теплового балансу, підвищення апетиту, підвищення збудливості та дратівливості, екзофтальм і тахікардія. Яке ендокринне порушення призводить до появи даних симптомів? In the course of the patient's clinical examination, an increase in the thyroid gland (goiter), an increase in the basal metabolism, a loss of body weight, a violation of the thermal balance, an increase in appetite, an increase in excitability and irritability, exophthalmos and tachycardia. What endocrine disorder leads to the appearance of these symptoms?

Гіперфункція щитоподібної залози Hyperthyroidism

Гіпофункція паращитоподібних залоз Hypofunction of parathyroid glands

Гіпофункція епіфізу Pineal hypofunction

Гіпофункція щитоподібної залози Hypothyroidism

Гіперфункція гіпофізу Hyperfunction of the pituitary gland

103 / 200
Пошкодження мозку призвело до порушення моторної функції мови. У якому відділі кори відбулося пошкодження? Brain damage resulted in a violation of the motor function of speech. In which part of the cortex did the damage occur?

Скронева ділянка кори Temporal cortex

Тім’яна ділянка кори Parietal cortex

Нижня лобна звивина Inferior frontal gyrus

Передня центральна звивина Anterior central gyrus

Потилична ділянка кори Occipital cortex

104 / 200
Під час бійки у чоловіка виникла зупинка серця внаслідок сильного удару у верхню ділянку передньої черевної стінки. Які з зазначених рефлексів спричинили зупинку серця? During the fight, the man suffered a cardiac arrest as a result of a strong blow to the upper part of the front abdominal wall. Which of the indicated reflexes caused the cardiac arrest?

Периферичні Peripheral

Парасимпатичні умовні Parasympathetic conditional

Парасимпатичні безумовні Parasympathetic unconditional

Симпатичні безумовні Lovely Unconditional

Симпатичні умовні Lovely conditionals

105 / 200
У пахвах людини виявлені дрібні (11,5 мм) сплющені у спинно-черевному напрямку, безкрилі кровосисні комахи. Їх личинки розвивалися тут же. Яке захворювання викликають ці комахи? Small (11.5 mm) flattened in the dorso-ventral direction, wingless blood-sucking insects were found in the armpits of a person. Their larvae developed there. What diseases do these insects cause ?

Чума Plague

Фтиріоз Phthyriosis

Поворотний тиф Typhoid

Сонна хвороба Sleeping sickness

Хвороба Чагаса Chagas disease

106 / 200
У людини під впливом іонізуючого випромінювання в крові зменшилася кількість гранулоцитів. Який механізм лежить в основі цих змін? The number of granulocytes in the blood of a person under the influence of ionizing radiation has decreased. What mechanism is the basis of these changes?

Розвиток аутоімунного процесу Development of autoimmune process

Підвищене руйнування лейкоцитів Increased destruction of leukocytes

Порушення виходу зрілих лейкоцитів з кісткового мозку Disturbance of the release of mature leukocytes from the bone marrow

Збільшений перехід гранулоцитів в тканини Increased migration of granulocytes into tissues

Пригнічення лейкопоезу Suppression of leukopoiesis

107 / 200
Хворому проведено лобектомію правої середньої частки легені. Які сегменти легені були уражені? The patient underwent lobectomy of the right middle lobe of the lung. Which lung segments were affected?

Верхівковий, передній Apical, anterior

Верхівково-задній та передній Top-posterior and anterior

Бічний та присередній Lateral and medial

Задній і бічний основні Back and side main

Присередній і передній основні Middle and front primary

108 / 200
Пацієнт доставлений до лікарні з симптомами запаморочення, сухості в роті, зіниці сильно розширену порушення акомодації, тахікардія, утруднення сечовипускання, атонія кишечнику. Передозування яким препаратом могло викликати дані симптоми? The patient was taken to the hospital with symptoms of dizziness, dry mouth, pupils, greatly dilated accommodative disorder, tachycardia, difficulty urinating, intestinal atony. What drug overdose could have caused these symptoms ?

Фуросемід Furosemide

Клофелін Clofelin

Атропіну сульфат Atropine sulfate

Каптоприл Captopril

Празозин Prazozin

109 / 200
У чоловіка 60-ти років, який страждає на хронічну кишкову непрохідність, посилюється гниття білків у товстому кишечнику. Підтвердженням цього процесу є: A 60-year-old man suffering from chronic intestinal obstruction has increased protein decay in the large intestine. Evidence of this process is:

Білірубінурія Bilirubinuria

Глюкозурія Glucosuria

Креатинурія Creatinuria

Гіперурікурія Hyperuricuria

!ндиканурія !ndikanuria

110 / 200
Хворий у непритомному стані доставлений бригадою швидкої допомоги до лікарні. Об’єктивно: рефлекси відсутні, періодично з’являються судоми, дихання нерівномірне. Після лабораторного обстеження було діагностовано печінкову кому. Нагромадження якого метаболіту є суттєвим для появи розладів центральної нервової системи? The patient was brought to the hospital in an unconscious state by an ambulance. Objectively: reflexes are absent, convulsions appear periodically, breathing is irregular. After a laboratory examination, liver disease was diagnosed to whom. The accumulation of which metabolite is essential for the appearance of disorders of the central nervous system?

Глутамін Glutamine

Сечовина Urea

Гістамін Histamine

Амоніак Ammonia

Білірубін Bilirubin

111 / 200
Внаслідок переливання несумісної крові за антигеном Rh у хворої виникла гемолітична жовтяниця. Який лабораторний показник крові підтверджує цей тип жовтяниці? Due to the transfusion of incompatible blood for the Rh antigen, the patient developed hemolytic jaundice. What laboratory blood test confirms this type of jaundice?

Зменшення вмісту кон’югованого білірубіну Decrease in conjugated bilirubin

Зменшення вмісту стеркобіліну Decreasing the content of stercobilin

Нагромадження некон’югованого білірубіну Unconjugated bilirubin accumulation

Зменшення вмісту некон’югованого білірубіну Reduction of unconjugated bilirubin

Нагромадження уробіліногену Urobilinogen accumulation

112 / 200
Хворий госпіталізований з травмою медіальної групи м’язів стегна. Які види рухів він НЕ зможе робити? The patient is hospitalized with an injury of the medial group of thigh muscles. What types of movements will he NOT be able to do?

Згинання стегна Flexion of hip

Супінація стегна Thigh supination

Приведення стегна Thigh adduction

Відведення стегна Thigh abduction

Розгинання стегна Extension of the hip

113 / 200
При обстеженні хворого з ендокринною патологією встановлено, що в плазмі крові підвищений рівень тестостерону. Які клітини в організмі чоловіка відповідальні за продукцію цього гормону? During the examination of a patient with endocrine pathology, it was established that the level of testosterone in the blood plasma is elevated. Which cells in a man's body are responsible for the production of this hormone?

Іландулоцити сім’яників Ilandulocytes of testes

Клітини сім’яних міхурців Seminal vesicle cells

Клітини передміхурової залози Prostate cells

Сустентоцити сім’яників Sustentocytes of testes

Сперматогенні клітини Spermatogenic cells

114 / 200
Курареподібні речовини (дитилін) роблять неможливим скорочення скелетних м’язів, оскільки вони блокують: Curare-like substances (dithyline) make skeletal muscle contraction impossible because they block:

Нервово-м’язові синапси Neuromuscular synapses

Іангліонарні синапси Ianglionic synapses

Центральні синапси Central synapses

Проведення збудження мембраною Conduction of excitation through the membrane

Пропріорецептори Proprioreceptors

115 / 200
Встановлено, що швидкість проведення збудження нервовими волокнами становить 120 м/сек. Зазначені волокна є: It was established that the speed of conduction of excitation by nerve fibers is 120 m/sec. The specified fibers are:

Постгангліонарними симпатичними Postganglionic sympathetic

Постгангліонарними парасимпатичними Postganglionic parasympathetic

Прегангліонарними симпатичними Preganglionic sympathetic

Прегангліонарними парасимпатичними Preganglionic parasympathetic

Аксонами мотонейронів Axons of motor neurons

116 / 200
У хворого при обстеженні виявлено глюкозурію, гіперглікемію. Скарги на сухість в роті, свербіння шкіри, часте сечовиділення, спрагу. Встановлений діагноз: цукровий діабет. Чим обумовлена поліурія у даного хворого? Glucosuria and hyperglycemia were detected in the patient during the examination. Complaints of dry mouth, itchy skin, frequent urination, thirst. The diagnosis was established: diabetes mellitus. What causes polyuria in this patient?

Зменшення серцевого викиду Decreasing cardiac output

Збільшення фільтраційного тиску Increase in filtration pressure

Збільшення осмотичного тиску сечі Increase in urine osmotic pressure

Збільшення онкотичного тиску плазми Increase in oncotic plasma pressure

Зменшення онкотичного тиску плазми Reduction of plasma oncotic pressure

117 / 200
При сильному зубному болі хворому рекомендовано прийняти ненар-котичний анальгетик із групи похідних аніліну. Виберіть препарат: For severe toothache, the patient is recommended to take a non-narcotic analgesic from the group of aniline derivatives. Choose a drug:

Ібупрофен Ibuprofen

Кислота ацетилсаліцилова Acetylsalicylic acid

Бутадіон Butadione

Анальгін Analgin

Парацетамол Paracetamol

118 / 200
В лікарню доставлена жінка, у якої виникла необхідність в інтубації трахеї. Який з нижченаведених лікарських засобів доцільно застосувати у даному випадку? A woman was brought to the hospital who needed tracheal intubation. Which of the following drugs should be used in this case?

Атропіну сульфат Atropine sulfate

Метронідазол Metronidazole

Нітрогліцерин Nitroglycerin

Іентаміцину сульфат Ientamycin sulfate

Дитилін Ditylin

119 / 200
Жінці, що страждає на алергічний нейродерміт, призначено антигістамін-ний препарат II покоління, в якого відсутній пригнічуючий вплив на ЦНС. Вкажіть цей препарат: A woman suffering from allergic neurodermatitis was prescribed an antihistamine drug of the II generation, which does not have a depressing effect on the central nervous system. Specify this drug:

Димедрол Diphenhydramine

Кетотіфен Ketotifen

Тавегіл Tavegil

Лоратадин Loratadine

Діазолін Diazolin

120 / 200
Судово-медичному експерту надійшов орган для ідентифікації. При гістологічному дослідженні в ньому виявлені невпорядковано розміщені лім-фоїдні фолікули, які мають 4 зони і ексцентрично розташовану в них артерію. Який це орган? The forensic medical expert received an organ for identification. Histological examination revealed irregularly placed lymphoid follicles, which have 4 zones and an eccentrically located artery in them. What organ is this?

Апендикс Appendix

Лімфатичний вузол Lymph node

Селезінка Spleen

Товста кишка Colon

Мигдалик Amygdalik

121 / 200
Хворий 37-ми років за останні три місяці схуд на 5 кг, скаржиться на тремор рук, підвищене потовиділення, екзофтальм, тахікардію. Збільшення секреції якого гормону може бути причиною цього? A 37-year-old patient has lost 5 kg of weight over the past three months, complains of hand tremors, increased sweating, exophthalmos, tachycardia. An increase in the secretion of which hormone can be the cause this?

Кортизол Cortisol

Тироксин Tyroxin

Інсулін Insulin

Тиреокальцитонін Thyrocalcitonin

Глюкагон Glucagon

122 / 200
У кішки під час експерименту подразнюють периферичний відрізок блукаючого нерва. Які з наведених змін будуть спостерігатися при цьому? During the experiment, the peripheral segment of the vagus nerve is irritated in a cat. Which of the following changes will be observed?

Збільшення частоти дихання Increased breathing rate

Розширення бронхів Bronchi dilatation

Зменшення частоти серцевих скорочень Decreasing heart rate

Збільшення частоти серцевих скорочень Increased heart rate

Розширення зіниць Pupil Dilation

123 / 200
Після того, як людина випила 1,5 л води, кількість сечі значно збільшилась, а її відносна щільність зменшилась до 1,00І. Зазначені зміни є наслідком зменшення реабсорбції води в дистальних відділах нефронів внаслідок зменшення секреції: After a person drank 1.5 liters of water, the amount of urine increased significantly, and its relative density decreased to 1.00I. These changes are the result of a decrease in reabsorption of water in the distal parts of the nephrons due to a decrease in secretion:

Ангіотензину II Angiotensin II

Реніну Renin

Простагландинів Prostaglandins

Вазопресину Vasopressin

Альдостерону Aldosterone

124 / 200
У хлопчика водянка яєчка (накопичування рідини між оболонками яєчка). Яка саме оболонка яєчка вміщує цю рідину? A boy has hydrocele (accumulation of fluid between the scrotum). Which scrotum contains this fluid?

М’ясиста Fleshy

Внутрішня сім’яна Internal seed

Зовнішня сім’яна External seed

Піхвова Vagina

Білкова Bilkova

125 / 200
При обстеженні хворого 6-ти років виникла підозра на погіршення прохідності дихальних шляхів. Який із методів дослідження дозволяє вірогідно визначити дану патологію? During the examination of a 6-year-old patient, there was a suspicion of deterioration of the patency of the respiratory tract. Which of the research methods allows you to reliably determine this pathology?

Спірометрія Spirometry

Спірографія Spirography

Пневмографія Pneumography

Спірометаболографія Spirometabolography

Пневмотахометрія Pneumotachometry

126 / 200
У чоловіка 60-ти років, який страждає на хронічний гепатит, часто спостерігались кровотечі з носа і ясен, спонтанно виникали геморагічні висипання на шкірі і слизових оболонках. Наслідком чого є ці явища? A 60-year-old man suffering from chronic hepatitis had frequent bleeding from the nose and gums, spontaneous hemorrhagic rashes on the skin and mucous membranes. As a result, are there these phenomena?

Зменшення утворення сироваткових альбумінів Decreased formation of serum albumins

Зменшення синтезу протромбіну і фібриногену Decreased synthesis of prothrombin and fibrinogen

Поява в крові макроглобулінів і кріоглобулінів Appearance of macroglobulins and cryoglobulins in the blood

Зменшення в крові рівня холінесте-рази Decrease in the level of cholinesterase in the blood

Підвищення вмісту амінотрансфераз плазми крові Increased blood plasma aminotransferases

127 / 200
Провідними симптомами первинного гіперпаратиреозу є остеопороз та ураження нирок із розвитком сечокам’яної хвороби. Які речовини складають основу каменів при цьому захворюванні? The leading symptoms of primary hyperparathyroidism are osteoporosis and kidney damage with the development of urolithiasis. What substances form the basis of stones in this disease?

Сечова кислота Uric acid

Холестерин Cholesterol

Фосфат кальцію Calcium phosphate

Цистін Cystine

Білірубін Bilirubin

128 / 200
У хворої встановлено порушення виділення тиреотропного гормону гіпофіза. Порушення функції якої частки гіпофіза має місце у хворої? The patient has been diagnosed with a violation of the release of thyroid-stimulating hormone from the pituitary gland. Which lobe of the pituitary gland has a dysfunction in the patient?

Lobus posterior Lobus posterior

Infundibulum Infundibulum

- -

Lobus anterior Lobus anterior

Pars intermedia Pars intermedia

129 / 200
У жінки 49-ти років, після тривалого стояння виявлено набряк ніг. Яка можлива причина появи набряків? A 49-year-old woman developed leg swelling after standing for a long time. What is the possible cause of swelling?

Підвищення артеріального тиску Increased blood pressure

Зменшення гідростатичного тиску крові у венах Reduction of hydrostatic blood pressure in veins

Підвищення гідростатичного тиску крові у венах Increased hydrostatic blood pressure in the veins

Зменшення гідростатичного тиску крові в артеріях Reduction of hydrostatic blood pressure in arteries

Збільшення онкотичного тиску плазми крові Increase in oncotic pressure of blood plasma

130 / 200
У хворого після прийому недоброякісної їжі розвинувся багаторазовий пронос. На наступний день у нього знизився артеріальний тиск, з’явилися тахікардія, екстрасистолія. рН крові 7,18. Ці порушення є наслідком: The patient developed repeated diarrhea after eating poor-quality food. The next day, his blood pressure decreased, tachycardia, extrasystole appeared. Blood pH 7.18. These the violation is a consequence of:

Негазового алкалозу Nongaseous alkalosis

Газового ацидозу Gas acidosis

Метаболічного алкалозу Metabolic alkalosis

Негазового ацидозу Nongaseous acidosis

Газового алкалозу Gas alkalosis

131 / 200
Хворий переведений на безсольову дієту. Як у нього змінився поріг смакової чутливості до солоного? The patient was transferred to a salt-free diet. How did his threshold of taste sensitivity to salty change?

Знизився Decreased

Підвищився Increased

Мало змінився Little changed

Спочатку підвищився, а потім знизився Initially increased, then decreased

Не змінився Has not changed

132 / 200
На аутопсії померлого, який три- вало страждав на гіпертонічну хворобу, патологоанатом виявив, що нирки різко зменшені у розмірах, щільні, поверхня їх рівномірно дрібнозерниста, на розрізі - паренхіма, особливо кіркова речовина, рівномірно виточені. Він дійшов висновку, що це: At the autopsy of the deceased, who suffered from hypertension for a long time, the pathologist found that the kidneys were sharply reduced in size, dense, their surface was uniformly fine-grained, on section - the par enchyma, especially the cortical substance, is uniformly extruded. He concluded that it:

Артеріолосклеротично зморщена нирка Arteriolosclerotic shrunken kidney

Пієлонефритично зморщена нирка Pyelonephritic shrunken kidney

- -

Амілоїдно-зморщена нирка Amyloid shrunken kidney

Атеросклеротично зморщена нирка Atherosclerotic shrunken kidney

133 / 200
Пацієнт звернувся до лікаря з приводу того, що він втратив здатність розрізняти смаки на корені язика. Лікар встановив, що це пов’язано з ураженням нерва. Якого саме? A patient consulted a doctor about the fact that he lost the ability to distinguish tastes at the root of the tongue. The doctor determined that this was due to nerve damage. Which one exactly?

Язикоглотковий Lingopharyngeal

Лицьовий Face

Трійчастий Triple

Верхньогортанний Upper laryngeal

Блукаючий Wandering

134 / 200
Хворий скаржиться на жовтяни-чність шкіри, шкірний свербіж, загальну слабкість. У сечі: відсутній уробілін. Яка патологія у хворого? The patient complains of jaundice of the skin, skin itching, general weakness. In the urine: there is no urobilin. What is the patient's pathology?

Гемолітична жовтяниця Hemolytic Jaundice

Гостра печінкова недостатність Acute liver failure

Паренхіматозна жовтяниця Parenchymal jaundice

Механічна жовтяниця Mechanical jaundice

Хронічна печінкова недостатність Chronic liver failure

135 / 200
Введення знеболюючого пацієнту перед екстракцією зуба призвело до розвитку анафілактичного шоку, який супроводжувався розвитком олігурії. Який патогенетичний механізм зумовив зменшення діурезу в даній клінічній ситуації? The administration of an anesthetic to a patient before tooth extraction led to the development of anaphylactic shock, which was accompanied by the development of oliguria. What pathogenetic mechanism caused the decrease in diuresis in this clinical situation?

Збільшення секреції вазопресину Increased vasopressin secretion

Збільшення онкотичного тиску плазми крові Increase in oncotic pressure of blood plasma

Підвищення гідростатичного тиску в капсулі Шумлянського-Боумена Increase in hydrostatic pressure in the Shumlyansky-Bowman capsule

Пошкодження клубочкового фільтру Damage of the glomerular filter

Зниження гідростатичного тиску в капілярах ниркових тілець Reduction of hydrostatic pressure in the capillaries of the renal corpuscles

136 / 200
Для формування мінерального ма-триксу твердих тканин зуба необхідна висока концентрація фосфат-іонів, що утворюється в процесі гідролізу фо-сфорноефірних зв’язків за участю лужної фосфатази. Іони якого металу є активаторами цього процесу: For the formation of the mineral matrix of the hard tissues of the tooth, a high concentration of phosphate ions is required, which is formed in the process of hydrolysis of phosphorus ester bonds with the participation of alkaline phosphatase. Ions which metal are the activators of this process:

Магній Magnesium

Натрій Sodium

Цинк Zinc

Кальцій Calcium

Ферум Ferum

137 / 200
При втручанні з метою лікування вивиху нижньої щелепи лікар повинен пам’ятати про м’яз, який при скороченні відтягує назовні капсулу і суглобовий диск скроневонижньощелепного суглоба. Який це м’яз? When intervening for the treatment of dislocation of the lower jaw, the doctor must remember the muscle that, when contracted, pulls the capsule and articular disc of the temporomandibular joint outward. What is this m 'I'm from?

M. temporalis M. temporalis

M. masseter M. masseter

M. pterygoideus medialis M. pterygoideus medialis

M. pterygoideus lateralis M. pterygoideus lateralis

M. mylohyoideus M. mylohyoideus

138 / 200
У чоловіка 71-го року впродовж 10 днів спостерігався пронос з домішками у калі слизу та крові. Хворого було госпіталізовано у тяжкому стані, помер через 2 доби. При розтині тіла померлого виявлено: дифтеритичний коліт з множинними виразками неправильної форми різної глибини в сигмоподібній і прямій кишках. При бактеріологічному дослідженні висіяно шигели. Яке основне захворювання у хворого? A 71-year-old man had diarrhea with impurities of mucus and blood in his stool for 10 days. The patient was hospitalized in serious condition, died 2 days later. At autopsy the body of the deceased revealed: diphtheritic colitis with multiple ulcers of irregular shape of different depths in the sigmoid and rectum. Bacteriological examination revealed Shigella. What is the patient's main disease?

Ієрсиніоз Yersiniosis

Черевний тиф Typhoid

Дизентерія Dysentery

Сальмонельоз Salmonellosis

Неспецифічний виразковий коліт Nonspecific ulcerative colitis

139 / 200
Зірчастий ретикулум і зовнішній шар клітин емалевого органу піддаються інволюції і після завершення аме-логенезу разом з апікальними частинами амелобластів формують: The stellate reticulum and the outer layer of cells of the enamel organ undergo involution and after completion of amelogenesis together with the apical parts of ameloblasts form:

- -

Муцинову плівку Mucin film

Зубний наліт (бляшки) Plaque

Пелікулу зуба Tooth pellicle

Кутикулу зуба Tooth cuticle

140 / 200
При мікроскопічному дослідженні біоптата з товстої кишки виявлена пухлина з призматичного епітелію, що формує атипові залозисті структури різної форми і величини. Базальна мембрана залоз зруйнована. Клітки пухлини поліморфні, ядра гіперхромні, відзначається велика кількість патологічних мітозів. Який діагноз найбільш вірогідний? During microscopic examination of a biopsy from the colon, a tumor of prismatic epithelium was found, which forms atypical glandular structures of various shapes and sizes. The basal membrane of the glands is destroyed. The tumor cells are polymorphic, the nuclei hyperchromic, a large number of pathological mitoses are noted. What is the most likely diagnosis?

Базальноклітинний рак Basal cell carcinoma

Недиференційований рак Undifferentiated cancer

Аденокарцинома Adenocarcinoma

Слизовий рак Mucosal cancer

Солідний рак Solid cancer

141 / 200
При мікроскопії мазка з плівки, яка з’явилась на пептонній воді через 6 годин після посіву фекалій і культивування в термостаті, виявлені зігнуті у вигляді коми рухливі бактерії, грам-негативні, спор і капсул не утворюють. Які це мікроорганізми? On microscopy of a smear from a film that appeared on peptone water 6 hours after inoculation of feces and cultivation in a thermostat, motile bacteria bent in the form of a comma were found, gram - negative, do not form spores and capsules. What are these microorganisms?

Вібріони Vibrios

Коринебактерії Corynebacteria

Спірили Spiril

Спірохети Spirochetes

Клостридії Clostridia

142 / 200
Для морфологічного дослідження представлена ендокринна залоза, паренхіма якої складається з епітелію та нервової тканини. В епітеліальних тра-бекулах виявляється 2 типи клітин: хро-мофільні та хромофобні. Визначте даний орган: An endocrine gland is presented for morphological examination, the parenchyma of which consists of epithelium and nervous tissue. 2 types of cells are found in epithelial trabeculae: chromophilic and chromophobic. Define this body:

Надниркова залоза Adrenal gland

Гіпофіз Pituitary

Прищитоподібна залоза Parathyroid gland

Гіпоталамус Hypothalamus

Щитоподібна залоза Thyroid

143 / 200
До приймального відділення надійшов хворий з ознаками гострої серцевої недостатності: блідість, акроціаноз, часте, поверхневе дихання. Який з перелічених засобів показаний у цьому випадку? A patient with signs of acute heart failure arrived at the reception department: paleness, acrocyanosis, frequent shallow breathing. Which of the listed remedies is indicated in this case?

Дигітоксин Digitoxin

Корглікон Corglycon

Адреналіну гідрохлорид Adrenaline hydrochloride

Кордіамін Cordiamine

Нітрогліцерин Nitroglycerin

144 / 200
В гістологічному препараті нижньої щелепи ембріону виявляється зубний зачаток, в якому зубний сосочок утворений дрібними зірчастими базо-фільно забарвленими клітинами. Яка тканина утворює цю частину зубного зачатку? In the histological preparation of the lower jaw of the embryo, a tooth bud is revealed, in which the dental papilla is formed by small stellate basophil-stained cells. What tissue forms this part of the tooth bud?

Хрящова Khryaschova

Мезенхіма Mesenchyme

Епітеліальна Epithelial

Ретикулярна Reticular

Кісткова Bone

145 / 200
За професійними показаннями проведена вакцинація лікарів- стоматологів. Вакцина має захищати їх від вірусної інфекції, збудник якої може бути присутній у крові стоматологічних хворих, які перенесли інфекцію або є хронічними носіями. Яка вакцина була використана? According to professional indications, vaccination of dentists was carried out. The vaccine should protect them from a viral infection, the causative agent of which may be present in the blood of dental patients who have suffered an infection or are chronic carriers. What vaccine was used?

Генно-інженерний НВs-антиген Genetically engineered HBs-antigen

!нактивована вакцина проти гепатиту А !activated hepatitis A vaccine

Жива корова вакцина Live cow vaccine

Субодинична грипозна вакцина Subunit influenza vaccine

Антирабічна вакцина Anti-rabies vaccine

146 / 200
У хворого виявлено порушення секреторної функції піднижньощелепної слинної залози. Який нерв забезпечує її вегетативну іннервацію? The patient has a violation of the secretory function of the submandibular salivary gland. What nerve provides its vegetative innervation?

N. petrosus major N. petrosus major

Chorda tympani Chorda tympani

N. auriculotemporalis N. auriculotemporalis

N. mandibularis N. mandibularis

N. petrosus minor N. petrosus minor

147 / 200
При огляді новонародженої дитини педіатр виявив наявність в ротовій порожнині нижніх центральних різців. В якому віці вони прорізуються в нормі? When examining a newborn child, the pediatrician found the presence of lower central incisors in the oral cavity. At what age do they erupt normally?

6-8 місяців 6-8 months

Впродовж першого місяця життя During the first month of life

До народження Before birth

До 2-х років Up to 2 years

10-12 місяців 10-12 months

148 / 200
У хворого зі скаргами на часті рідкі випорожнення з кров’ю ('малинове желе') при мікроскопічному дослідженні були виявлені великі клітини з одним ядром та поглиненими еритроцитами. Для якого з найпростіших характерна така морфологічна форма? In a patient with complaints of frequent liquid stools with blood ('raspberry jelly'), microscopic examination revealed large cells with a single nucleus and absorbed erythrocytes. For which of the simplest organisms is characterized by such a morphological form?

Giardia lamblia Giardia lamblia

Entamoeba histolytica Entamoeba histolytica

Campylobacter jejuni Campylobacter jejuni

Balantidium coli Balantidium coli

Toxoplasma gondii Toxoplasma gondii

149 / 200
У жінки 30-ти років з деформацією нижньої щелепи вилучена пухлина у вигляді вузла. Макроскопічно тканина її бура, з порожнинами; мікроскопічно представлена дрібними одноядер-ними клітинами овальної форми за типом остеобластів та гігантськими багатоядерними клітинами за типом остео-кластів, серед них багато дрібних незрілих судин, зерна гемосидерину, острівці остеоїдної речовини. Який найбільш імовірний діагноз? A 30-year-old woman with a deformity of the lower jaw had a tumor in the form of a node removed. Macroscopically, her tissue is brown, with cavities; microscopically, it is represented by small oval-shaped uninucleate cells by the type of osteoblasts and giant multinucleated cells by the type of osteoclasts, among them many small immature vessels, grains of hemosiderin, islands of osteoid matter. What is the most likely diagnosis?

Остеосаркома Osteosarcoma

Гемангіома Hemangioma

Фіброзна остеодисплазія Fibrous osteodysplasia

Остеобластокластома Osteoblastoclastoma

Остеома Osteoma

150 / 200
Хворому на остеомієліт нижньої щелепи був призначений антибіотик тетрациклінової групи. Вкажіть цей препарат: A patient with osteomyelitis of the lower jaw was prescribed an antibiotic of the tetracycline group. Specify this drug:

Оксацилін Oxacillin

Доксицикліну гідрохлорид Doxycycline hydrochloride

Рифампіцин Rifampicin

Стрептоміцин Streptomycin

Амікацин Amikacin

151 / 200
При огляді ротової порожнини на вестибулярній поверхні нижнього різця зліва виявлене утворення грибоподібної форми рожевого кольору до 2 см, яке широкою ніжкою фіксоване до надальвеолярної тканини. Під час гістологічного дослідження виявлено розгалужені судини капілярного типу з судинними бруньками, ділянками крововиливів та осередки гемосидерозу. Який найбільш імовірний діагноз? When examining the oral cavity on the vestibular surface of the lower incisor on the left, a mushroom-shaped formation of pink color up to 2 cm was found, which was fixed to the supraalveolar tissue with a wide stem. During the histological examination, it was found branched vessels of the capillary type with vascular buds, areas of hemorrhage and foci of hemosiderosis. What is the most likely diagnosis?

Гігантоклітинний епуліс Giant cell epulis

Ангіоматозний епуліс Angiomatous epulis

Кавернозна гемангіома Cavernous hemangioma

Фіброзний епуліс Fibrous epulis

Фіброматоз ясен Gingival fibromatosis

152 / 200
Пацієнту з метою підтримання роботи печінки був призначений препарат, якій містить метіонін. Завдяки цьому може бути забезпечено синтез: A drug containing methionine was prescribed to the patient in order to maintain liver function. Thanks to this, the synthesis can be ensured:

Лактату Lactate

Пірувату Pyruvate

Фосфатидилхоліну Phosphatidylcholine

Фосфатидилсерину Phosphatidylserine

Цитрату Citrate

153 / 200
Жінці 26-ти років, хворій на бронхіт, призначили засіб етіотропної терапії - антибіотик широкого спектру дії. Який це препарат? A 26-year-old woman with bronchitis was prescribed a means of etiotropic therapy - a broad-spectrum antibiotic. What is this drug?

Амброксол Ambroxol

Дексаметазон Dexamethasone

БЦЖ-вакцина BCG vaccine

Доксициклін Doxycycline

Інтерферон Interferon

154 / 200
У хворого герпетичний кон’юнктивіт. Який етіотропний лікарський засіб необхідно призначити? The patient has herpetic conjunctivitis. What etiotropic drug should be prescribed?

Метисазон Metisazone

Ампіцилін Ampicillin

Тетрациклін Tetracycline

Фурагін Furagin

Ацикловір Acyclovir

155 / 200
Пацієнт 68-ми років звернувся до кардіолога із скаргами на підвищення артеріального тиску, біль в ділянці серця, відчуття перебоїв у роботі серця. Призначте препарат з групи в1-адреноблокаторів для лікування даної патології: A 68-year-old patient turned to a cardiologist with complaints of increased blood pressure, pain in the heart area, a feeling of interruptions in the work of the heart. Prescribe a drug from the group of b1-adrenoblockers for the treatment of this pathology:

Бензилпеніцилін Benzylpenicillin

Морфіну гідрохлорид Morphine hydrochloride

Метопролол Metoprolol

Ноотропіл Nootropil

Фенотерол Fenoterol

156 / 200
Жінка під час вагітності хворіла на краснуху. Дитина народилась з вадами розвитку - незрощення губи і піднебіння. Генотип у дитини нормальний. Ці аномалії розвитку є проявом: The woman had rubella during pregnancy. The child was born with malformations of the lip and palate. The child's genotype is normal. These developmental abnormalities are a manifestation of:

Хромосомної мутації Chromosomal mutation

Поліплоїдії Polyploids

Модифікаційної мінливості Modification variability

Анеуплоїдії Aneuploidy

Комбінативної мінливості Combinative variability

157 / 200
У спекотну погоду в гарячих приміщеннях для нормалізації мікроклімату часто використовують вентилятори. При цьому посилюється віддача тепла тілом людини перш за все таким тттля-хом: In hot weather, fans are often used in hot rooms to normalize the microclimate. At the same time, the return of heat from the human body increases, first of all, like this:

Кондукція Conduct

Випаровування Evaporation

Теплопроведення Heat conduction

Радіація Radiation

Конвекція Convection

158 / 200
Вагітній з кількома мимовільними абортами в анамнезі призначено терапію вітамінними препаратами. Який вітамін сприяє виношуванню плода? A pregnant woman with a history of several spontaneous abortions is prescribed vitamin therapy. Which vitamin helps to carry the fetus?

Ціанокобаламін Cyanocobalamin

Тіамін Thiamine

Альфа-токоферол Alpha-tocopherol

Піридоксальфосфат Pyridoxal phosphate

Рутин Routine

159 / 200
З носоглотки здорового бакте-ріоносія виділено мікроорганізм, який за біологічними ознаками ідентичний Corynebacterium diphtheriae, але не продукує екзотоксин. В результаті якого процесу цей мікроорганізм здатний відновити експресію і продукцію екзото- ксину? A microorganism was isolated from the nasopharynx of a healthy carrier, which is biologically identical to Corynebacterium diphtheriae, but does not produce an exotoxin. As a result of what process is this microorganism able to restore expression and production exotoxin?

Хромосомна мутація Chromosome mutation

Культивування на телуритовому середовищі Cultivation on tellurite medium

Пасаж через організм чутливих тварин Passage through the body of sensitive animals

Фагова конверсія Phage conversion

Культивування в присутності антитоксичної сироватки Cultivation in the presence of antitoxic serum

160 / 200
Оперативно висічена сполучна тканина деформованого мітрального клапану при забарвленні гематоксиліном та еозином дає базофільну реакцію, а під впливом толуїдинового синього фарбується в бузковий колір (ме-тахромазія). Які зміни сполучної тканини виявляються такими типами реакцій? Operatively excised connective tissue of a deformed mitral valve when stained with hematoxylin and eosin gives a basophilic reaction, and under the influence of toluidine blue it turns lilac (metachromasia). What are the changes of connective tissue are revealed by the following types of reactions?

Набряк сполучної тканини Swelling of connective tissue

Гіаліноз Hyalinosis

Фібриноїдний некроз сполучної тканини Fibrinoid necrosis of connective tissue

Мукоїдний набряк Mucoid edema

Петрифікація Petrification

161 / 200
Внаслідок надмірного годування тварини вуглеводами в клітинах печінки при гістологічному дослідженні виявлена значна кількість гранул глікогену. До якої групи структур клітини відноситься глікоген? As a result of excessive feeding of the animal with carbohydrates, a significant number of glycogen granules were found in the liver cells during histological examination. To which group of cell structures does glycogen belong?

Секреторні включення Secretary inclusions

Трофічні включення Trophic inclusions

Пігментні включення Pigment inclusions

Екскреторні включення Excretory inclusions

Органели спеціального призначення Special purpose organs

162 / 200
У хворого виражений больовий синдром при невралгії. Який засіб з не-стероїдних протизапальних препаратів зменшить болесприйняття? The patient has a pronounced pain syndrome with neuralgia. Which non-steroidal anti-inflammatory drug will reduce pain perception?

Диклофенак-натрій Diclofenac sodium

Кодеїну фосфат Codeine Phosphate

Лідокаїну гідрохлорид Lidocaine hydrochloride

Кетаміну гідрохлорид Ketamine hydrochloride

Дроперидол Droperidol

163 / 200
У хворого розвинувся птоз (опущення верхньої повіки). Ураження якого нерва має місце? The patient developed ptosis (lowering of the upper eyelid). Which nerve is affected?

Блоковий Block

Лицевий Face

Трійчастий Triple

Відвідний Deferred

Окоруховий Okoruhovy

164 / 200
Стеатоз виникає внаслідок нако- пичення триацилгліцеролів у гепатоци-тах. Одним з механізмів розвитку цього захворювання є зменшення утилізації нейтрального жиру ЛПДНЩ. Які ліпо-тропні речовини попереджують розвиток стеатозу? Steatosis occurs as a result of the accumulation of triacylglycerols in hepatocytes. One of the mechanisms of the development of this disease is a decrease in the utilization of neutral VLDL fat. What lipotropic substances prevent the development of steatosis ?

Аланін, B1, PP Alanine, B1, PP

Аргінін, B2, B3 Arginine, B2, B3

Метіонін, B6, B12 Methionine, B6, B12

Валін, B3, B2 Valin, B3, B2

Iзолейцин, B1, B2 Isoleucine, B1, B2

165 / 200
До медико-генетичної консультації звернувся чоловік з кольоровою сліпотою (дальтонізм). З якою ймовірністю у його дітей виявиться дана ознака, якщо в генотипі його дружини даного алеля немає? A man with color blindness (colorblindness) turned to medical and genetic counseling. What is the probability that his children will have this trait, if this allele is not present in his wife's genotype?'

100% 100%

75% 75%

50% 50%

0% 0%

25% 25%

166 / 200
Велика доза сечогінного засобу викликала у хворого падіння артеріального тиску. Яку групу засобів найбільш доцільно використати для його підвищення у цьому випадку? A large dose of a diuretic caused a drop in blood pressure in the patient. What group of drugs is most appropriate to use to increase it in this case?

Серцеві глікозиди Cardiac glycosides

Н-холіноміметики H-cholinomimetics

Плазмозамінники Plasma substitutes

Аналептики Analeptics

Адреноміметики Adrenomimetics

167 / 200
В клітині відбувається процес трансляції. Коли рибосома доходить до кодонів УАА, УАГ або УГА, синтез по-ліпептидного ланцюга закінчується. Ці кодони у процесі біосинтезу поліпептиду не розпізнаються жодною т-РНК і тому є сигналом: The translation process takes place in the cell. When the ribosome reaches the UAA, UAG or UGA codons, the synthesis of the polypeptide chain ends. These codons in the process of polypeptide biosynthesis are not recognized by any t-RNA is therefore a signal:

Початку транскрипції Start of transcription

Ініціації Initiations

Елонгації Elongations

Посттрансляційної модифікації Post-translational modification

Термінації Terminations

168 / 200
Після початку лікування туберкульозу легень, хворий звернувся до лікаря зі скаргами на появу червоних сліз та сечі. Який препарат міг викликати такі зміни? After starting treatment for pulmonary tuberculosis, the patient turned to the doctor complaining of the appearance of red tears and urine. What drug could have caused such changes?

Бензилпеніциліну натрієва сіль Benzylpenicillin sodium salt

Рифампіцин Rifampicin

Бісептол-480 Biseptol-480

Бензилпеніциліну калієва сіль Benzylpenicillin potassium salt

Цефазолін Cefazolin

169 / 200
У хворого запальний процес у кри-лопіднебінній ямці. !нфекція поширилась в носову порожнину. Через яке анатомічне утворення розповсюдилась інфекція? The patient has an inflammatory process in the pterygoid fossa. The infection has spread to the nasal cavity. Through which anatomical structure did the infection spread?

Canalis ptherygoideus Canalis ptherygoideus

Canalis palatinus minor Canalis palatinus minor

Canalis palatinus major Canalis palatinus major

Foramen rotundum Foramen rotundum

Foramen sphenopalatinum Foramen sphenopalatinum

170 / 200
У хворого запальний процес у крилопіднебінній ямці. !нфекція поширилась в середню черепну ямку. Через яке анатомічне утворення розповсюдилась інфекція? The patient has an inflammatory process in the pterygopalatine fossa. The infection has spread to the middle cranial fossa. Through which anatomical formation has the infection spread?

Canalis palatinus minor Canalis palatinus minor

Foramen rotundum Foramen rotundum

Foramen sphenopalatinum Foramen sphenopalatinum

Canalis ptherygoideus Canalis ptherygoideus

Canalis palatinus major Canalis palatinus major

171 / 200
Гістологічне дослідження тканини виявило, що в ній відсутні кровоносні судини, а клітини щільно прилягають одна до одної, утворюючи пласти. Яка це тканина? The histological examination of the tissue revealed that there are no blood vessels in it, and the cells are closely adjacent to each other, forming layers. What kind of tissue is this?

м’язова muscular

кісткова bone

епітеліальна epithelial

нервова nervous

хрящова cartilage

172 / 200
При гістологічному дослідженні мікропрепарату шкіри людини виявляється тільки щільна неоформлена сполучна тканина. Який шар даного органу був представлений для вивчення? During histological examination of a micropreparation of human skin, only dense, irregular connective tissue is detected. What layer of this organ was presented for study?

Сосочковий шар дерми Papillary layer of the dermis

Епідерміс Epidermis

Сітчастий шар дерми Reticular layer of the dermis

Підшкірна жирова клітковина Subcutaneous adipose tissue

Базальний шар епідермісу Basal layer of the epidermis

173 / 200
Після ретельного обстеження хворого, який повернувся із Середньої Азії до України, було встановлено попередній діагноз: весняно-літній енцефаліт. Через укус якого членистоногого збудник міг потрапити до організму? After a thorough examination of a patient who returned from Central Asia to Ukraine, a preliminary diagnosis was established: spring-summer encephalitis. Through the bite of which arthropod could the pathogen enter the body?

Селищний кліщ House tick

Коростяний свербун Scabies itch

Москіт Mosquito

Тайговий кліщ Taiga tick

Собачий кліщ Dog tick

174 / 200
Хворому було поставлено попередні дiагноз: парагонімоз. Ця хвороба спричиняється легеневим сисуном. Збудник потрапив до організму хворого під час: The patient was given a preliminary diagnosis: paragonimosis. This disease is caused by a lung fluke. The causative agent entered the patient's body during:

Вживання в їжу немитих овочів Eating unwashed vegetables

Вживання в їжу напівсирих раків i крабів Eating semi-raw crayfish and crabs

Пиття сирої води з відкритих водойм Drinking raw water from open water

Вживання в їжу напівсирої чи в’яленої риби Eating semi-raw or dried fish

Контакту з хворою кішкою Contact with a sick cat

175 / 200
Після обстеження хворого лікар рекомендував йому вилучити з раціону наваристі м’ясні та овочеві бульйони, прянощі, копчені продукти, оскільки у хворого було виявлено: After examining the patient, the doctor recommended that he remove rich meat and vegetable broths, spices, and smoked products from his diet, as the patient was found to have:

Зменшення моторики шлунково-кишкового тракту Decreased motility of the gastrointestinal tract

Дискінезія жовчних шляхів Biliary dyskinesia

Зменшення слиновиділення Decreased salivation

Зменшення секреції хлористоводневої кислоти залозами шлунка Decreased secretion of hydrochloric acid by the gastric glands

Збільшення секреції хлористоводневої кислоти залозами шлунка Increased secretion of hydrochloric acid by the gastric glands

176 / 200
У хворого відмічені ознаки атеросклерозу. Вміст яких транспортних форм ліпідів підвищений у плазмі крові хворого? The patient has signs of atherosclerosis. The content of which transport forms of lipids is increased in the patient's blood plasma?

ЛППЩ LDL

ЛПНЩ LDL

ЛПДНЩ LPDNS

Хіломікрони Chylomicrons

ЛПВЩ HDL

177 / 200
З метою перевірки якості чистоти повітря в операційній було проведено висів повітря на ряд поживних середовищ за допомогою апарата Кротова. Після інкубування на деяких з них виросли колонії, оточені чіткою зоною гемолізу. Які середовища було використано для перевірки повітря? In order to check the quality of air cleanliness in the operating room, air was sown on a number of nutrient media using the Krotov apparatus. After incubation, some of them grew colonies surrounded by a clear zone hemolysis. What media was used to test the air?

Сироватковий МПА Serum MPA

Середовище Ендо Environment Endo

Кров’яний МПА Blood MPA

Жовтково-сольовий агар Yolk-salt agar

Вісмутсульфіт агар Bismuth Sulfite Agar

178 / 200
Лікар-гематолог призначив пацієнту з кровотечею коагулянт, який діє шляхом підвищення синтезу протромбіну та інших факторів зсідання крові переважно в печінці, є синтетичним водорозчинним вітаміном. Який препарат призначив лікар? A hematologist prescribed a coagulant to a patient with bleeding, which acts by increasing the synthesis of prothrombin and other blood clotting factors mainly in the liver, is a synthetic water-soluble vitamin. What drug did the doctor prescribe ?

Вікасол Vikasol

Етамзилат Etamsylate

Кальцію хлорид Calcium chloride

Тромбін Thrombin

Гепарин Heparin

179 / 200
У хворого порушена функція м’язів гомілки. Він не може утримати своє тіло піднявшись навшпиньки. Функція якого м’яза постраждала? The patient has impaired leg muscle function. He cannot hold his body while standing on tiptoes. Which muscle function is affected?

M. extensor digitorum longus M. extensor digitorum longus

M. tibialis posterior M. tibialis posterior

M. flexor digitorum longus M. flexor digitorum longus

M. tibialis anterior M. tibialis anterior

M. triceps surae M. triceps surae

180 / 200
Екзотоксин дифтерійної палички обробили 0,3-0,4% формаліном і витримали 30 днів у термостаті при температурі 40oC. Який препарат був отриманий у результаті проведених маніпуляцій? The diphtheria bacillus exotoxin was treated with 0.3-0.4% formalin and kept for 30 days in a thermostat at a temperature of 40oC. What drug was obtained as a result of the manipulations?

Анатоксин Anatoxin

Діагностична сироватка Diagnostic serum

Діагностикум Diagnostics

Антитоксин Antitoxin

Лікувальна сироватка Healing Serum

181 / 200
Після отриманої травми хворий не може змістити нижню щелепу в бік та висунути її вперед. Функція яких м’язів імовірно була пошкоджена? After the injury, the patient cannot shift the lower jaw to the side and push it forward. Which muscle function was probably damaged?

Скроневі Temporal

Щелепно-під’язикові Maxillohyoid

Щічні Cheeks

Жувальні Chewing

Бічні крилоподібні Lateral pterygoids

182 / 200
У хворого 30-ти років на нижній губі виявлена виразка із гладенькою лакованою поверхнею, червоного кольору, хрящоподібної консистенції. Біопсія місця ураження виявила лімфоплазмо-цитарну інфільтрацію, явища васкуліту. Який найбільш імовірний діагноз? In a 30-year-old patient, an ulcer with a smooth lacquered surface, red color, cartilage-like consistency was found on the lower lip. A biopsy of the affected area revealed lymphoplasmacytic infiltration, phenomena of vasculitis . What is the most likely diagnosis?

Гума Rubber

Вісцеральний сифіліс Visceral syphilis

Вторинний сифіліс Secondary syphilis

Первинний сифіліс Primary syphilis

Вроджений сифіліс Congenital syphilis

183 / 200
До відділення хірургічної стоматології потрапив чоловік з травматичним розривом arcus venosus jugulae, яка знаходиться в spatium: A man was admitted to the department of surgical dentistry with a traumatic rupture of the arcus venosus jugulae, which is located in the spatium:

Interaponeuroticum suprasternale Interaponeuroticum suprasternale

Previscerale Previscerale

Antescalenum Antescalenum

Retropharyngeale Retropharyngeale

Interscalenum Interscalenum

184 / 200
Аналіз ЕКГ хворого виявив відсутність зубця P. Тривалість та амплітуда комплексу QRS та зубця T відповідають нормі. Що є водієм ритму серця даного пацієнта? Analysis of the patient's ECG revealed the absence of a P wave. The duration and amplitude of the QRS complex and the T wave correspond to the norm. What is the driver of this patient's heart rhythm?

Пучок Пса Dog Bunch

Передсердно-шлуночковий вузол Atrioventricular node

Синусний вузол Sine Node

Волокна Пуркін’є Purkinje fibers

Міокард шлуночків Myocardium of ventricles

185 / 200
Деякі білки слини виконують захисну функцію. Який з них захищає слизову оболонку ротової порожнини від механічних ушкоджень? Some saliva proteins perform a protective function. Which of them protects the mucous membrane of the oral cavity from mechanical damage?

Каталаза Catalase

Муцин Mucin

Пероксидаза Peroxidase

Ренін Renin

Лізоцим Lysozyme

186 / 200
У пацієнта для іонофорезу при бактеріальному періодонтиті застосували спиртовий розчин йоду. Вкажіть механізм терапевтичної дії цього засобу: An alcohol solution of iodine was applied to the patient for iontophoresis with bacterial periodontitis. Specify the mechanism of therapeutic action of this agent:

Заміна атомів водню при атомі азоту в аміногрупі білка Replacement of hydrogen atoms at nitrogen atom in the amino group of the protein

Відновлення нітрогрупи під впливом нітроредуктаз Reduction of the nitro group under the influence of nitroreductases

Утворення альбумінатів Formation of albuminates

Гальмування утворення клітинної стінки Inhibition of cell wall formation

Зміна поверхневого натягу мембрани мікробної клітини Change in the surface tension of the microbial cell membrane

187 / 200
Чоловік повернувся з Лівану. Через деякий час він відчув біль і важкість у промежині і надлобковій ділянці. При обстежені йому було встановлено діагноз - урогенітальний шистосо-моз. Яким шляхом він міг заразитися? The man returned from Lebanon. After some time, he felt pain and heaviness in the perineum and suprapubic area. During the examination, he was diagnosed with urogenital schistosomiasis. In which way could he get infected?

Недосмажене м’ясо великої рогатої худоби Uncooked beef

Недоварене м’ясо раків і крабів Uncooked crayfish and crab meat

Через немиті овочі та фрукти Because of unwashed vegetables and fruits

Недостатньо просолена риба Undersalted fish

Купання у заражених водоймах Swimming in contaminated water bodies

188 / 200
Лікар виконує передній серединний розтин шиї для термінової трахео- томії. Про можливе пошкодження якої судини він повинен пам’ятати? The doctor performs an anterior median neck dissection for an urgent tracheotomy. Which vessel should he be aware of for possible damage?

Vfacialis Vfacialis

Vjugularis interna Vjugularis interna

Arcus venosus juguli Arcus venosus juguli

V.jugularis externa V. jugularis externa

Vthyroidea media Vthyroidea media

189 / 200
До приймального відділення надійшов хворий з важкою щелепно-лицевою травмою. Який препарат йому необхідно ввести для зняття больового шоку? A patient with a severe maxillofacial injury was admitted to the reception department. What drug should be administered to him to relieve pain shock?

Промедол Promedol

Пантогам Pantogamus

Мідокалм Midocalm

Сиднокарб Sydnokarb

Ібупрофен Ibuprofen

190 / 200
При обробці фрезою великого кутнього зуба стоматолог інструментом, що зірвався, глибоко поранив щоку, пошкодивши при цьому не лише слизову оболонку але й м’яз. Який? While processing a large canine tooth with a milling cutter, the dentist deeply injured the cheek with a broken tool, damaging not only the mucous membrane but also the muscle. Which one?

Великий виличний м’яз Major zygoma muscle

Коловий м’яз рота Orbital muscle

Щічний Buccal

Жувальний Chewing

Щелепно-під’язиковий м’яз Maxillohyoid muscle

191 / 200
Під впливом іонізуючого опромінення або при авітамінозі Е в клітині спостерігається підвищення проникності мембран лізосом. До яких наслідків може призвести така патологія? Under the influence of ionizing radiation or vitamin E deficiency, an increase in the permeability of lysosome membranes is observed in the cell. What consequences can this pathology lead to?

Відновлення цитоплазматичної мембрани Cytoplasmic membrane restoration

Інтенсивний синтез енергії Intensive energy synthesis

Інтенсивний синтез білків Intensive protein synthesis

Часткове чи повне руйнування клітини Partial or complete cell destruction

Формування веретена поділу Formation of division spindle

192 / 200
При лікуванні хворого на спадкову форму імунодефіциту було застосовано метод генотерапії: ген ферменту був внесений у клітини пацієнта за допомогою ретровірусу. Яка властивість генетичного коду дозволяє використовувати ретровіруси у якості векторів функціональних генів? In the treatment of a patient with a hereditary form of immunodeficiency, the method of gene therapy was used: the enzyme gene was introduced into the patient's cells using a retrovirus. What property of the genetic code allows retroviruses to be used as vectors functional genes?

Безперервність Continuity

Універсальність Versatility

Колінеарність Colinearity

Надмірність Redundancy

Специфічність Specificity

193 / 200
Провідну роль в процесі кальцифі- кації тканин зуба відіграє білок осте-окальцин, який має високу здатність зв’язувати іони кальцію, завдяки наявності в поліпептидному ланцюзі залишків модифікованої амінокислоти: The leading role in the process of tooth tissue calcification is played by the osteocalcin protein, which has a high ability to bind calcium ions due to the presence of modified amino acid residues in the polypeptide chain :

Гама-карбоксиглутамінова Gamma-carboxyglutamine

Іама-аміномасляна Iama-amino-butter

Аланін Alanine

Дельта-амінопропіонова Delta-aminopropion

Карбоксиаспарагінова Carboxysparagin

194 / 200
Лікар виявив у дитини рахіт, зумовлений нестачею вітаміну D, але за своїм проявом подібний до спадкового вітаміностійкого рахіту (викривлення трубчастих кісток, деформація суглобів нижніх кінцівок, зубні абсцеси). Як називаються вади розвитку, які нагадують спадкові, але не успадковуються? The doctor discovered rickets in the child caused by a lack of vitamin D, but its manifestation is similar to hereditary vitamin-resistant rickets (curvature of tubular bones, deformation of the joints of the lower limbs, dental abscesses) . What is the name of developmental defects that resemble hereditary ones, but are not inherited?

Моносомії Monosomy

Фенокопії Phenocopies

Генокопії Genocopies

Іенні хвороби Yearly diseases

Трисомії Trisomies

195 / 200
У хворого 69-ти років на шкірі в ділянці нижньої повіки з’явилося невелике бляшкоподібне утворення, з послі-дуючим виразкуванням, яке було оперативно видалене. При мікроскопічному дослідженні утворення: в дермі шкіри комплекси з атипових епітеліальних клітин, на периферії утворень клітини розташовані перпендикулярно до базальної мембрани. Клітини темні, призматичної полігональної форми; ядра гіперхромні з частими мітозами. Іноді зустрічаються утворення, подібні до волосяного фолікула. Яка гістологічна форма рака у хворого? In a 69-year-old patient, a small plaque-like formation appeared on the skin in the area of the lower eyelid, with subsequent ulceration, which was surgically removed. Upon microscopic examination formation: in the dermis of the skin are complexes of atypical epithelial cells, on the periphery of the formations, cells are located perpendicular to the basement membrane. Cells are dark, prismatic polygonal in shape; nuclei are hyperchromic with frequent mitoses. Sometimes there are formations similar to a hair follicle. What is the histological form of cancer in the patient?

Базально-клітинний Basal cell

Аденокарцинома Adenocarcinoma

Плоскоклітинний без ороговіння Squamous without cornification

Недиференційований Undifferentiated

Плоскоклітинний з ороговінням Squamous with keratinization

196 / 200
Жінка 35-ти років хворіє на ВІЛ-інфекцію на стадії СНІД. На шкірі нижніх кінцівок, слизової оболонки піднебіння з’явились рудувато-червоні плями, яскраво-червоні вузлики різних розмірів. Один з вузликів взято на гістологічне дослідження. Виявлено багато хаотично розташованих тонко- стінних судин, вистелених ендотелієм; пучки веретеноподібних клітин з наявністю гемосидерину. Яка пухлина розвинулась у хворої? A 35-year-old woman has HIV infection at the AIDS stage. Reddish-red spots appeared on the skin of the lower extremities, the mucous membrane of the palate, bright red nodules of different sizes. One of the nodules was taken for histological examination. Many thin-walled vessels lined with endothelium were found in a chaotic manner; bundles of spindle-shaped cells with the presence of hemosiderin. What tumor developed in the patient?

Лімфома Беркіта Burkitt's lymphoma

Фібросаркома Fibrosarcoma

Лімфангіома Lymphangioma

Саркома Капоші Kaposi's sarcoma

Гемангіома Hemangioma

197 / 200
Хворий звернувся до лікаря з гострим зубним болем в нижньому лівому іклі. Поставлено діагноз - пульпіт. Який нерв забезпечує чутливість цього зуба? The patient went to the doctor with a sharp toothache in the lower left canine. A diagnosis of pulpitis was made. What nerve provides the sensitivity of this tooth?

Нижній комірковий Bottom cell

Виличний High

Піднебінний Palate

Верхній комірковий Upper cell

Лицевий Face

198 / 200
Відомо, що стероїдні протизапальні препарати гальмують активність фо-сфоліпази А2, що необхідна для синтезу простагландинів. Яка речовина є попередником цих медіаторів запалення? Steroid anti-inflammatory drugs are known to inhibit the activity of phospholipase A2, which is necessary for the synthesis of prostaglandins. What substance is the precursor of these inflammatory mediators?

Тирозин Tyrosine

Холестерол Cholesterol

Пальмітинова кислота Palmitic acid

Проопіомеланокортин Proopiomelanocortin

Арахідонова кислота Arachidonic acid

199 / 200
В стоматологічній практиці для дослідження збудливості тканин зубів використовують метод електроодонтоді-агностики. При цьому визначається: In dental practice, the method of electroodontodiagnosis is used to study the excitability of tooth tissues. In this case, the following is determined:

Корисний час Useful time

Хронаксія Chronaxis

Поріг сили подразника Threshold of stimulus strength

Акомодація Accommodation

Лабільність Lability

200 / 200
Студент перед іспитом скаржився на гострий зубний біль, який послабився під час складання іспиту. Яке гальмування зумовило зменшення больових відчуттів? Before the exam, the student complained of a sharp toothache, which subsided during the exam. What inhibition caused the pain to decrease?

Згасаюче Fading

Диференціювальне Differentiating

Запізніле Late

Зовнішнє External

Позамежне Foreign